contracts mnemonics · 2019. 9. 23. · ©2018 pieper bar review 1 . contracts mnemonics . 1) the...

66
©2018 Pieper Bar Review 1 CONTRACTS MNEMONICS 1) The ingredients for a valid contract are TACO: T Definite TERMS, express or implied A – ACCEPTANCE of terms C – CONSIDERATION O – OFFER inviting acceptance 1a) Eat a PEAR to recover in quasi contract: P – Plaintiff’s good faith PERFORMANCE of services, E – Plaintiff’s EXPECTATION of compensation; A – ACCEPTENCE of the services performed; and R – For the recovery of REASONABLE value of the services performed 2) An offer expires when it gets TIRED: T Reasonable TIME after an offer is made, or after expiration date expressly stated in an offer I Mental INCAPACITY or death of offeror or offeree R – REVOCATION of an offer communicated to an offeree before acceptance E – EXPRESS or implied rejection communicated to offeror D – DESTRUCTION of the subject matter of the offer or intervening illegality terminates an offer by operation of law 3) Options can DIE by: D – DESTRUCTION of subject matter I – Intervening ILLEGALITY E – EXPIRATION of a stated option time extinguishes the option 4) Generally, contracting parties are free to modify a 3 rd party beneficiary (3PB) K, unless, prior to receiving notice of the K modification, the 3PB got MAD: M – MANIFESTED an assent called for in the 3PB K, at the request of one of the contracting parties (i.e., accepted a K offer arising from the 3PB K) A Commenced a breach of K ACTION against the promisor, or D – DETRIMENTALLY relied on the K 5) Contract assignments may involve the ADA: A – ASSIGNMENT of a contractual right to collect money owed under the K D – DELEGATION of the performance required under the K A – ASSUMPTION of liability for performing the K

Upload: others

Post on 13-Aug-2021

3 views

Category:

Documents


0 download

TRANSCRIPT

Page 1: CONTRACTS MNEMONICS · 2019. 9. 23. · ©2018 Pieper Bar Review 1 . CONTRACTS MNEMONICS . 1) The ingredients for a valid contract are TACO: T – Definite TERMS, express or implied

©2018 Pieper Bar Review 1

CONTRACTS MNEMONICS 1) The ingredients for a valid contract are TACO:

T – Definite TERMS, express or implied A – ACCEPTANCE of terms C – CONSIDERATION O – OFFER inviting acceptance

1a) Eat a PEAR to recover in quasi contract: P – Plaintiff’s good faith PERFORMANCE of services, E – Plaintiff’s EXPECTATION of compensation; A – ACCEPTENCE of the services performed; and R – For the recovery of REASONABLE value of the services performed 2) An offer expires when it gets TIRED:

T – Reasonable TIME after an offer is made, or after expiration date expressly stated in an offer

I – Mental INCAPACITY or death of offeror or offeree R – REVOCATION of an offer communicated to an offeree before acceptance E – EXPRESS or implied rejection communicated to offeror D – DESTRUCTION of the subject matter of the offer or intervening illegality

terminates an offer by operation of law 3) Options can DIE by:

D – DESTRUCTION of subject matter I – Intervening ILLEGALITY E – EXPIRATION of a stated option time extinguishes the option

4) Generally, contracting parties are free to modify a 3rd party beneficiary (3PB) K, unless, prior to

receiving notice of the K modification, the 3PB got MAD: M – MANIFESTED an assent called for in the 3PB K, at the request of one of the

contracting parties (i.e., accepted a K offer arising from the 3PB K) A – Commenced a breach of K ACTION against the promisor, or D – DETRIMENTALLY relied on the K

5) Contract assignments may involve the ADA:

A – ASSIGNMENT of a contractual right to collect money owed under the K D – DELEGATION of the performance required under the K A – ASSUMPTION of liability for performing the K

Page 2: CONTRACTS MNEMONICS · 2019. 9. 23. · ©2018 Pieper Bar Review 1 . CONTRACTS MNEMONICS . 1) The ingredients for a valid contract are TACO: T – Definite TERMS, express or implied

©2018 Pieper Bar Review 2

6) A gratuitous assignment becomes irrevocable, and a second assignee prevails over a prior assignee of

the contract when J.P.N.C.: J – Recovers a JUDGMENT P – Gets PAID, or NC – Enters a NEW CONTRACT with the obligator

7) Absent express language in a K prohibiting assignment, K rights are freely assignable, except those of SIR: S – Where a STATUTE expressly prohibits the assignment of a K right (but if that

claim is reduced to judgment, it is assignable) I – Where the assignment is coupled with an IMPROPER delegation of a duty

under the K to a person unqualified to fulfill that duty R – Where the assignment increased the RISK to the other contracting party

8) Statues prohibit assignment of a WASP:

W – WORKER’S COMPENSATION payments A – ALIMONY or child support payments S – SPENDTHRIFT TRUSTS income P – PERSONAL INJURY or wrongful death clamp

9) Look at HAIL to determine whether a breach is material or immaterial:

H – HARDSHIP on breaching party if total material breach is declared A – AMOUNT of benefit bestowed on non–breaching party I – Whether breach was INNOCENT L – LIKELIHOOD of full performance being achieved

10) Breach of contract defenses are I3 FU2MED & I S2IP:

I – INFANCY I – INSANITY – INCOMPETENCY I – INTOXICATION F – FRAUD U – UNCONSCIONABILITY U – UNDUE INFLUENCE M – MISTAKE E – EQUITABLE DEFENSES D – DURESS I – IMPOSSIBILITY of performance S – STATUTE OF FRAUDS S – STATUTE OF LIMITATIONS I – ILLEGALITY P – PAROLE EVIDENCE RULE

11) Lack of contractual capacity arises from the 3 I’s: I – INFANCY I – INTOXICATION I – Mental INFIRMITY

Page 3: CONTRACTS MNEMONICS · 2019. 9. 23. · ©2018 Pieper Bar Review 1 . CONTRACTS MNEMONICS . 1) The ingredients for a valid contract are TACO: T – Definite TERMS, express or implied

©2018 Pieper Bar Review 3

12) SI2R M is a fraud: S – SCIENTER I – D lied with an INTENT to defraud the P I – P suffered an economic INJURY R – P justifiably RELIED on D’s misrepresentation M – D misrepresented a MATERIAL fact, which induced P to enter the K

13a)Badges of constructive fraud SHIRTS:

S – SECRETLY done H – HASTILY done I – INADEQUACY of consideration R – A close RELATIONSHIP between the transferor and transferee T – TRANSFEROR continues to the control property S – SCIENTIFIC (knowledge) of the creditor’s claim, an inability to pay it after

the transfer.

13) A unilateral mistake in calculating figures may allow the mistaken party the remedy of equity of rescission, if he calls the COPS: C – The computational mistake was COMMUNICATED to the other party before

that person changed his/her position in reliance on those mistaken figures O – The mistake involved was one of ORDINARY negligence P – The mistaken party gave PROMPT notice of the mistake S – The mistake will impose SUBSTANTIAL hardship on the party if not corrected

14) The following MRS SLY contracts must be in writing, subscribed by the party to be charged with the

breach (i.e., must contain defendant’s signature): M – MARRIAGE contracts R – REAL ESTATE contracts S – SURETY contracts S – UCC Article 2 SALES CONTRACTS for $500 or more L – LEASES longer than 1 year Y – Contracts not capable of complete performance within 1 YEAR

15) INTENTIONALLY LEFT BLANK FOR A FUTURE MNEMONIC

16) There are 4 T–CUP elements for a constructive trust: T – TRANSFER of property in reliance on promise C – Existence of CONFIDENTIAL or fiduciary relationship U – UNJUST enrichment to transferee of property or to some third party, AND P – PROMISE, express or implied, to hold property for plaintiff’s benefit, which

promise has been breached

Page 4: CONTRACTS MNEMONICS · 2019. 9. 23. · ©2018 Pieper Bar Review 1 . CONTRACTS MNEMONICS . 1) The ingredients for a valid contract are TACO: T – Definite TERMS, express or implied

©2018 Pieper Bar Review 4

17) A THUG may render an illegal contract enforceable, based on: T – TYPE of illegality & extent to which the public is harmed H – HARM that forfeiture would cause if contract was declared unenforceable due

to illegality; ct looks to see whether contract has been substantially performed U – UNJUST enrichment (a windfall) to party asserting illegality defense G – Relative GUILT of each party

18) The theory of impossibility frequently involves the 4 Ds:

D – DEATH D – DANGER to life/ill health D – DESTRUCTION of the subject matter of the law suit D – DELAYS, temporarily causing performance to become impracticable or

impossible 19) OF MICE2 permits parole evidence:

O – To establish an ORAL condition precedent to legal effectiveness of contract, provided it doesn’t contradict express term(s) of the contract

F – A party cannot invoke the Parole Evidence Rule to shield that party from allegations of FRAUD or Misrepresentation

M – To establish MUTUAL Mistake or claim for reformation of contract I – To establish ILLEGALITY C – To establish failure of CONSIDERATION E – To EXPLAIN ambiguous or missing terms E – To show that no ENFORCEABLE agreement was ever intended

20) Contract law does not allow damages recovery for CAPS:

C – To recover consequential damages, unless they were within the CONTEMPLATION OF BOTH PARTIES when the contract was executed

A – Damages that party could have AVOIDED P – Damages for PAIN & suffering or emotional distress resulting from a

breached contract, even if such damages were foreseeable S – SPECULATIVE damages aren’t recoverable (all damages must be proven

within a reasonable certainty) 21) Generally, parties can put whatever terms they’d like into a K, except for PLUS:

P – Terms that violate PUBLIC POLICY L – Terms providing for an excessive amount of LIQUIDATED DAMAGES U – Terms that are UNCONSCIONABLE S – Clauses providing that one party can seek SPECIFIC PERFORMANCE in

the event of a breach (the contract does NOT have to enforce these clauses)

22) Apply a TISSUE to a covenant restricting a former employee from competing: T – TIME restriction must be reasonable (usually two years or fewer) I – INABILITY of the employee to gain work elsewhere S – The geographic SPACE/SCOPE of the restriction must be as narrow as

possible (must only be to the extent necessary to protect employer’s interest) SUE – The employee services must be SPECIAL, UNIQUE, or EXTRAORDINARY

Page 5: CONTRACTS MNEMONICS · 2019. 9. 23. · ©2018 Pieper Bar Review 1 . CONTRACTS MNEMONICS . 1) The ingredients for a valid contract are TACO: T – Definite TERMS, express or implied

©2018 Pieper Bar Review 5

SALES MNEMONICS

23) ICOP limits the Perfect Tender Rule: I – INSTALLMENT contracts C – Timely delivery was COMMERCIALLY IMPRACTIBLE by an event not

contemplated by the parties. O – Delivery in good faith, OBJECTIVELY and reasonably believing the goods

would be acceptable to the buyer P – PRIOR TO DELIVERY DATE set forth in the contract, conforming goods

are delivered to replace the nonconforming goods 24) Additional terms will not be added to the contract when OCAN:

O – The offeror OBJECTS to additional terms within a reasonable time C – The offer expressly CONDITIONS the agreement on accepting the terms in

the offer as they are A – The additional terms materially ALTER the offer N – Either or both parties are NON–MERCHANTS

25) A J STRAW clause materially alters an offer if it would cause surprise or hardship to the offeror if the offeror was not made aware of its existence:

J – Bestowing JURISDICTION on a particular court, or requiring offeror to consent to jurisdiction in particular state

S – Shortening the STATUTE OF LIMITATIONS to sue for non–conforming goods

T – Limiting TORT liability or limiting a buyer’s right to sue for consequential damages R – Altering UCC rules for RISK OF LOSS A – Adding an ARBITRATION CLAUSE (unless customary to do so in the

trade) W – Adding a clause negating a WARRANTY (e.g., one of merchantability or

fitness) 26) Exceptions to the Statute of Frauds requirement are SWAMP:

S – Contracts for SPECIALLY manufactured goods W – WAIVER A – Judicial ADMISSION of contract M – “MERCHANT MEMORANDUM” P – PART PERFORMANCE

27) If a sales contract is silent on a topic, the UCC implies the following CIDER rules: C – Seller is not obligated to extend CREDIT to the buyer I – Buyer has the right to INSPECT the seller’s tendered goods (except no right

to inspect when the transaction involves a bill of lading) D – Seller’s tender of DELIVERY is implied to be at seller’s place of business,

unless both parties know that the goods are located elsewhere E – Buyer and seller must EXCHANGE performance concurrently R – RISK OF LOSS is on the party in the best position to bear that risk

Page 6: CONTRACTS MNEMONICS · 2019. 9. 23. · ©2018 Pieper Bar Review 1 . CONTRACTS MNEMONICS . 1) The ingredients for a valid contract are TACO: T – Definite TERMS, express or implied

©2018 Pieper Bar Review 6

28) SOAL–V and SORE–V affect risk of loss: SOAL V – SALE ON APPROVAL LATE VESTING (goods held by the

buyer are not subject to claims of the buyer’s creditors) SORE V – SALE OR RETURN EARLY VESTING (title and ROL vest

immediately in the buyer, even though the buyer has a right to rescind the K)

29) Remedies available to a seller are SPARKLE:

S – STOPPING goods in transit P – Suing for the entire contract PRICE A – Demanding ASSURANCES R – RE–SELLING goods to another buyer K – KEEPING part of a breaching buyer’s deposit, never more than $500 L – Suing for LOST Profit E – EXERCISING the right to reclaim goods delivered to the insolvent buyer

30) Remedies available to a buyer are CID’S WAR:

C – COVER I – INCIDENTAL & consequential damages D – DAMAGES for lost benefit of the bargain, or for the price paid S – SPECIFIC PERFORMANCE on a contract for unique goods W – Breach of WARRANTY A – ACCEPTANCE revocation R – REJECTING non–conforming goods

31) A sales contract contains M FEET warranties:

M – Warranty of MERCHANTABILITY F – Warranty of FITNESS for a particular purpose E – Warranty against ENCUMBRANCES E – EXPRESS warranties T – Warranty of TITLE

32) Express warranties are SAD:

S – SAMPLE or model, which is the basis of the bargain A – Written or oral AFFIRMATION of fact or promise made by the seller relating

to the goods D – DESCRIPTION of the goods in advertisements, brochures, or catalogs

33) A P’s claim against a seller for a defective good can be based on one or more overlapping but different PINE theories of liability: P – Torts theory of strict PRODUCTS liability I – Contract theory for breach of IMPLIED warranty N – Torts theory of NEGLIGENCE E – Contract theory for breach of an EXPRESS warranty

Page 7: CONTRACTS MNEMONICS · 2019. 9. 23. · ©2018 Pieper Bar Review 1 . CONTRACTS MNEMONICS . 1) The ingredients for a valid contract are TACO: T – Definite TERMS, express or implied

©2018 Pieper Bar Review 7

34) G.P.S. LAMP can use the following defenses against a breach of warranty claim: G – GOVERNMENT military contract defense P – Federal PREEMPTION S – STATUTE OF LIMITATIONS L – LACK of timely notice to a seller A – ASSUMPTION of risk (can be asserted against any PINE claim) M – Unforeseen MISUSE of a product P – Lack of PRIVITY of contract

35) Strict products liability is imposed on a regular seller of a DUD product:

D – DEFECTIVE, and an U – UNREASONABLY D – DANGEROUS product

36) When asserting a strict products liability claim, P must prove that a DIM dangerous defect in the product proximately caused a physical injury: D – DESIGN defect I – INADEQUATE warning M – Mistake in the MANUFACTURING process

Page 8: CONTRACTS MNEMONICS · 2019. 9. 23. · ©2018 Pieper Bar Review 1 . CONTRACTS MNEMONICS . 1) The ingredients for a valid contract are TACO: T – Definite TERMS, express or implied

©2018 Pieper Bar Review 8

Consideration Problems 1. An uncle promised his nephew, “If you refrain from drinking, smoking, and gambling until your

21st birthday, I’ll pay you $5,000.” The nephew agreed and then fulfilled his promise. Can the nephew enforce the promise? Yes. Even though there was no clear benefit flowing to the uncle, the nephew surrendered his rights to drink, gamble, and smoke based on the promise of $5,000 from the uncle. Therefore, there was valid consideration and the contract was enforceable. Hamer v. Sidway, 124 N.Y. 538 (1891).

2. A father tells his son, “I will pay you $10,000 when I receive my bonus, because I have not saved as much money in your 529 college savings account as I have saved in your siblings’ accounts.” Can the son force the father to pay? No. The father never requested or induced any promise or performance by the son. His statement was simply a promise to make a gift, which is not enforceable.

3. An employee saves her boss’s life in an emergency, and the employee becomes totally and permanently disabled in the process. Two months later, the boss promises to pay the employee $1,000 per month for the rest of her life. Is the boss’s promise enforceable? No. In the great majority of states, there would be no consideration for the promise to pay, since the benefit received was not induced by the promise to pay. However, a small minority of states would enforce the promise under the “Material Benefit Rule” outlined in Restatement 2nd of Contracts, Section 86. Under this rule, “the recipient of benefits who had no legal duty to pay for them may decide to promise payment for the past benefits received” and such promise will be enforced. John Edward Murray, Jr., Murray on Contracts, § 68[3], p. 327 (5th ed. 2011).

4. A farmer promised to pay her neighbor $5,000 at the end of the month for the neighbor’s tractor unless the farmer “changed her mind.” Can the contract be enforced? No. The promise is illusory, because the farmer has not committed herself to do anything. This is different from a conditional obligation. For example, if the farmer promised to pay the neighbor $5,000 for the neighbor’s tractor if the farmer was able to secure a loan at 5% interest or lower. Such a promise is, conditional, not illusory and would be enforceable.

5. If a restaurant and a website company entered a contract that allowed either party to terminate on 30 days’ notice, would the contract be enforceable? Yes, since the agreement obligated both parties for at least 30 days.

6. A builder entered a contract with a landowner to construct a home for $500,000. As the builder attempted to construct the foundation, she encountered solid bedrock that was unforeseen when the parties entered the contract and doubled the cost of construction. The builder notified the landowner of the problem and the landowner agreed to pay the increased cost totaling $1 million.

Page 9: CONTRACTS MNEMONICS · 2019. 9. 23. · ©2018 Pieper Bar Review 1 . CONTRACTS MNEMONICS . 1) The ingredients for a valid contract are TACO: T – Definite TERMS, express or implied

©2018 Pieper Bar Review 9

Upon completion of the home, is the landowner obligated to pay the $1 million?

No, in states that strictly follow the common law pre-existing duty rule. The original contract obligated to build the home on the property, performance was not impossible or impracticable, and the landowner received no additional consideration for the promise to pay twice the cost.

However, some jurisdictions following the Restatement position would enforce the promise, since the increase in price was made in good faith and fair under the circumstances.

7. An employee lost his foot in a job-related accident. He returned to his job almost a year later

and did not to sue his employer based upon an oral promise of lifetime employment. He was laid off the following year and sued for breach of the employment contract. Was there valid consideration for the agreement?

Yes. Even though the employee had no valid claim against the employer because his exclusive remedy was through worker’s compensation, if the employee was surrendering a claim in “good faith” a court would find valid consideration for the agreement. “Sufficient consideration requires more than the bald assertion by a claimant who has a claim, and to the extent that the validity or invalidity of a claim has a bearing upon whether there were reasonable grounds for believing in its possible validity, evidence of the validity or invalidity of a claim may be relevant to the issue of good faith.” Dyer v. National By-Products, Inc., 380 N.W.2d 732 (Iowa, 1986).

Page 10: CONTRACTS MNEMONICS · 2019. 9. 23. · ©2018 Pieper Bar Review 1 . CONTRACTS MNEMONICS . 1) The ingredients for a valid contract are TACO: T – Definite TERMS, express or implied

©2018 Pieper Bar Review 10

Damages The Goal: Contract law seeks to provide the non–breaching party with its Expectation Damages, which place the non–breaching party in as good a position as if the breaching party had performed its obligations under the contract. Expectation Damages are the usual form of damages awarded when a contract is breached.

The General Formula:

The Value of the Promised Performance MINUS

The Value of what the Plaintiff Received PLUS

Any Incidental and/or Consequential Damages MINUS

Any Costs that Were/Could have been Avoided/Mitigated

To make use of this formula, it helps to know something about each of its component parts: The “value of the promised performance” is typically just the contract price, though a fact pattern could tell you that the value of the performance was something else, for example, if the buyer was getting a good deal/discounted price. By allowing a plaintiff to recover the value of the promised performance as opposed to just the contract price, the plaintiff’s “benefit of the bargain” is preserved. The “value of what the plaintiff received” is exactly what it says: some value associated with the defendant’s performance (this value will be identified in the fact pattern, if necessary). “Incidental damages” are any “costs incurred in a reasonable effort, whether successful or not, to avoid loss, as where a party pays brokerage fees in arranging or attempting to arrange a substitute transaction.” Restatement 2d Contracts § 347, comment c. In a sale of goods contract, incidental damages are awarded to a non–breaching buyer for any reasonable expenses arising out of the breach including those incurred through cover, or any cost “reasonably incurred in inspection, receipt, transportation and care and custody of goods rightfully rejected . . . .” UCC 2–715. A non–breaching seller can recover incidental damages for charges incurred in “stopping delivery, in the transportation, care and custody of goods after the buyer's breach, [or] in connection with return or resale of the goods” etc. UCC 2–710. “Consequential damages” are additional losses incurred by the plaintiff as a result of the defendant’s breach, that usually arise in the area of lost profits following the defendant’s failure to perform on time. To recover consequential damages, the plaintiff must show 1) causation (the damages were a result of the defendant’s breach), 2) the damages were foreseeable at the time the parties entered into the contract, 3) a reasonable certainty as to the amount of damages, and 4) that the damages could not have been mitigated.

Page 11: CONTRACTS MNEMONICS · 2019. 9. 23. · ©2018 Pieper Bar Review 1 . CONTRACTS MNEMONICS . 1) The ingredients for a valid contract are TACO: T – Definite TERMS, express or implied

©2018 Pieper Bar Review 11

“Costs that were or could have been avoided or mitigated” are simply 1) costs that the plaintiff will no longer have to incur following the defendant’s breach (sometimes referred to as “costs avoided”), like any amounts that the plaintiff no longer has to pay under the breached contract, and 2) losses that the plaintiff can mitigate “by making substitute arrangements for the use of his [or her] resources that are no longer needed to perform the contract” (sometimes called “losses avoided”) Restatement 2d Contracts § 347, comment d. To avoid pitfalls, always remember that contract law prohibits the recovery of CAPS: C – CONSEQUENTIAL DAMAGES, unless they were within the contemplation of both parties (foreseeable) when the contract was executed A – Damages that party could have AVOIDED P – Damages for PAIN & suffering or emotional distress resulting from a breached contract, even if such damages were foreseeable S – SPECULATIVE damages (all damages must be proven within a reasonable certainty.

The Caveat: The formula and its component principles are a guide. Any damages formula will work well in some situations, but not work well in others. Our job is to think about concepts like placing the non–breaching party in as good a position as performance would have put her in, and not awarding damages for costs that could have been avoided, and then apply them as rationally as possible. By working through the scenarios to follow, you should gain an understanding of how the courts award damages in a range of different situations, so that you’ll be able to answer any damages questions on the bar exam.

Alternate Method of Looking at Damages: Scholars have developed an alternate measure of damages (not yet embraced by the courts but tested on the Multistate Bar Exam, see e.g. OPE 3 Q98) which breaks down the plaintiff’s recovery into expectation, restitution, and reliance interests. See Joseph M. Perillo, Calamari and Perillo on Contracts 490 (6th ed. 2009). Expectation Interest = the plaintiff’s expected gain under the contract (essentially lost profits) Restitution Interest = benefits conferred on the defendant that the plaintiff is entitled to recover Reliance Interest = the economic detriment incurred by the plaintiff as a result of the breached contract, which typically includes the restitution interest. This alternate, modern method embraces the same goal of placing the non–breaching party in as good a position as if the breaching party had performed its obligations under the contract, but uses these interests to categorize the elements of the plaintiff’s recovery. These terms are based on the traditional measures of damages (reliance and restitution).

Page 12: CONTRACTS MNEMONICS · 2019. 9. 23. · ©2018 Pieper Bar Review 1 . CONTRACTS MNEMONICS . 1) The ingredients for a valid contract are TACO: T – Definite TERMS, express or implied

©2018 Pieper Bar Review 12

Examples: PROBLEM #1: Waste Damages for Immaterial Breach = Cost of Completion (Unless Waste) When there has been a substantial performance (“HAIL”) in good faith, but a defect exists (especially one which is only incidental to the main purpose of the K), the a court will usually award the cost of completion to the non–breaching party (see Problem #2 above). However, if the cost to correct this minor deficiency is drastically large in proportion to the overall contract price, such that completion to the exact terms of the contract would constitute “economic waste,” the court will award an alternate measure of damages to protect the immaterially breaching party. This alternate measure of damages is the difference between the value of the property as constructed and the value of the property if performance had been properly completed. For example: A homeowner hired a contractor to build a home for $100,000 and the specifications required, among other things, that the pipes be “galvanized, lap welded pipe of the grade known as 'standard pipe' of Reading manufacture.” The contractor completed the house perfectly, other than that he mistakenly used nearly identical pipe manufactured, not by the Reading Manufacturing Company, but by the Cohoes Rolling Mill Company. The contractor and homeowner agreed that the contractor had substantially performed the contract and that the breach was immaterial, but the homeowner sought damages in the amount of $40,000 (the cost to complete the contract, which would require ripping–up and then refinishing large portions of the house). Here, the courts would say that replacing the Cohoes pipe with identical Reading pipe (the only difference being a stamp on the exterior of the pipe) would constitute economic waste. Therefore, the courts would award as an alternate measure of damages: the difference in value between the house with Cohoes pipe that was received, and the same house with Reading pipe that was called for in the contract. Jacobs & Young v. Kent, 230 N.Y. 239 (1921); Joseph M. Perillo, Contracts § 14.29 at 552-55 (7th ed 2014). However, courts will not award this alternate measure of damages just because the cost of completion is high. If the defect was not incidental to the contract, and the breaching party did not finish the contract, the cost of completion will be awarded even though the difference in value damages might be lower. For example, the owner of a 26–acre industrial site entered a contract with a demolition contractor to sell the scrap metal from its buildings and equipment for $275,000. The contract also required the demolition contractor to re–grade the property to make the property more suitable for resale. The demolition contractor removed the buildings and equipment, but failed to re–grade the property, and the owner sued for breach of contract. The owner sought the ordinary measure of damages, i.e. the cost of completion (the cost to re–grade the property), which was $110,000. The demolition contractor argued that this was waste, offering proof that the plaintiff–owner could sell the un–graded property for only $3,000 less than if the property was re–graded (difference in value damages). The court ruled that the demolition contractor owed the $110,000 cost of completion for the breach, noting “[Defendant–contractor’s] completed performance would not have involved undoing what in good faith was done improperly, but only doing what was promised and left undone.” American Standard, Inc. v. Schectman, 80 A.D.2d 318 (4th Dep’t 1981).

Page 13: CONTRACTS MNEMONICS · 2019. 9. 23. · ©2018 Pieper Bar Review 1 . CONTRACTS MNEMONICS . 1) The ingredients for a valid contract are TACO: T – Definite TERMS, express or implied

©2018 Pieper Bar Review 13

PROBLEM #2: Consequential Damages Must Be Foreseeable A carpenter entered a $10,000 contract to renovate a homeowner’s bathroom by July 1. Based on this contract, the carpenter entered a separate contract to buy a car for $10,000, to be delivered on July 2. The car contract provided that if payment for the car was not made on July 2, the cost of the car would increase to $12,000. The carpenter renovated the homeowner’s bathroom but was not timely paid. Consequently, the carpenter could not pay for the car on July 2, and was required to pay the additional $2,000 for the car at a later date. The carpenter cannot recover the additional $2,000 from the homeowner, because the breaching party is only liable for those consequential losses that were foreseeable based on the available information at the time of contracting. PROBLEM #3: Construction Contract Where Homeowner Breaches No Recovery for costs that could have been avoided A builder agreed to build a house for $100,000. When the builder had completed $80,000 of the project, the homeowner repudiated the contract and said she would not pay the builder. The builder then completed construction of the home, incurring an additional $5,000 cost. What can the builder recover from the homeowner? Consider that the contract price was $100,000, the builder’s reliance interest was $85,000, and his expectation interest was $15,000, but we don’t award damages for “CAPS” (the mnemonic above). A) $100,000 B) $ 95,000 C) $ 80,000 D) $ 85,000 It cost the builder $85,000 to construct a $100,000 house. From these figures, a court can determine that the builder would have made a profit of $15,000 (his expectancy interest). However, the builder cannot recover the $5,000 expended after the homeowner repudiated the contract, because under contract law a party who stubbornly continues to perform after the other party has repudiated cannot recover losses that could have been avoided ($5,000). Thus, the answer is B, $95,000, which is the value of what was promised ($100,000), less what was received ($0), plus any consequential or incidental damages ($0), less the $5,000 cost that could have been avoided by stopping work after the repudiation. Under the alternate, modern approach, the amount needed to make the homeowner whole would still be $95,000, consisting of an $80,000 reliance interest for costs incurred prior to the repudiation, plus the builder’s $15,000 lost profit expectancy interest. As with the explanation above, the builder cannot recover costs that could have been avoided.

Page 14: CONTRACTS MNEMONICS · 2019. 9. 23. · ©2018 Pieper Bar Review 1 . CONTRACTS MNEMONICS . 1) The ingredients for a valid contract are TACO: T – Definite TERMS, express or implied

©2018 Pieper Bar Review 14

PROBLEM #4: Emotional Distress Arising Out of a Breached Contract A builder agreed to construct a house for a homeowner. The builder knew when the contract was made that the homeowner was in delicate health and the new house was of great importance to her. When the house was done, the homeowner inspected it while the builder waited outside. When the homeowner came out, she slammed the front door and the whole house collapsed. Can the homeowner recover for her emotional distress because of the builder’s breach of contract? No. Such damages ordinarily are not allowed in a breach of contract action. Keep in mind that there are rare exceptions to this rule, where the court determines that a severe emotional disturbance was not only foreseeable, but a likely result of a breach. The limited circumstances in which a court has awarded damages for a severe emotional disturbance arising out of a breached contract include breached contracts for the burial of a family member, where a messenger is aware of the contents of a death notification and fails to make a timely delivery resulting in a relative missing a funeral, where a person agrees to be filmed for television on condition that her face is blacked out but his face is shown, etc. John Edward Murray, Jr., Murray on Contracts, § 124 (5th ed. 2011). PROBLEM #5: Breach by Paying Party Construction Contract Where Homeowner Breaches 5(a). A builder agreed with a homeowner to build a house for $100,000, which would have generated a $10,000 profit for the builder. If the homeowner repudiated the contract before the builder began, then the builder could sue for his lost bargain of $10,000, which is the value of the performance (here, the contract price of $100,000), minus what the builder received (here, nothing, since the fact pattern gives no indication that the homeowner was paid anything), minus the cost of completion (here, the $90,000 expenditure avoided by the builder not having to complete performance). Under the alternate, modern method, the damages would be the same, but the $10,000 would categorized as the plaintiff’s expectancy interest. 5(b). If the builder had partially performed the contract, spending $60,000 on construction prior to the homeowner’s repudiation (anticipatory breach), the builder’s damages would be $70,000, i.e., the value of the performance (here, the contract price of $100,000), minus what the builder received (here, nothing, since the fact pattern gives no indication that the homeowner was paid anything), minus the cost of completion (here, the remaining $30,000 expenditure avoided by the builder not having to complete performance). Under the alternate, modern approach, the amount needed to make the builder whole would still be $70,000, consisting of a $60,000 reliance interest, plus a $10,000 expectancy interest. 5(c). If the builder had completed the contract, spending $90,000, then he would be entitled to the entire contract price. That could be measured by the value of the performance (here, the contract price of $100,000), minus what the builder received (here, nothing, since the fact pattern gives no indication that the homeowner was paid anything), minus the cost of completion (here, nothing, since the house was completed). Under the alternate, modern approach, the amount needed to make the builder whole would still be $100,000, consisting of a $90,000 reliance interest, plus a $10,000 expectancy interest.

Page 15: CONTRACTS MNEMONICS · 2019. 9. 23. · ©2018 Pieper Bar Review 1 . CONTRACTS MNEMONICS . 1) The ingredients for a valid contract are TACO: T – Definite TERMS, express or implied

©2018 Pieper Bar Review 15

PROBLEM #6: Repudiation Prior to Performance Repudiation by Painter on Discounted Performance A painter contracted with a homeowner to paint the homeowner’s house. The value of such a paint job was $1,200, but the painter needed work and agreed to do the job for $900. Before the homeowner paid any money to the painter, the painter then repudiated the contract. 6(a). If the homeowner is then forced to hire another painter to complete the job for $1,100 (a price reflecting a $100 discount for what we’ve been told was the value of performance), the homeowner cannot recover $900 from the repudiating painter, but only the loss incurred as a result of entering a substitute contract. That is, the homeowner contracted to receive a painted house for $900. As a result of the breach, she received the painted house for $1,100. Since she is entitled to be placed in as good a position as if the first painter performed (a painted house for $900), the court will award her damages of $200 to reflect the difference between what she paid as a result of the breach ($1,100) and the original contract price ($900). 6(b). If after the painter’s repudiation, the homeowner decided not to paint her house, she is still entitled to damages her “lost bargain.” That is, contract law allows her to recover as if she had decided to hire another painter. The law sets the price of this substitute transaction at the market price (which the parties can establish through expert testimony). So in this situation, the homeowner would be entitled to recover the difference between the market value of the job ($1,200) and what the homeowner would have had to pay the painter under the contract ($900), resulting in an expectation damages claim of $300 for the homeowner’s lost bargain. Another way of looking at the same problem would be to award the homeowner the value of the performance ($1,200), less the value of what was received ($0), less the cost of paying the painter $900 (a cost avoided), for a total of $300. 6(c). If the painter painted the homeowner’s house defectively (the fact pattern would have to tell you the performance was worth, for example, only $700), then the homeowner’s damages in the event she did not hire someone to fix the paint job, are similar to the scenario above, $500 (the difference between the value of the properly performed paint job ($1,200) and the value of what the homeowner received (a $700 paint job)). PROBLEM #7: Construction Contract Where Builder Breaches Owner Recovers the Cost of Completion 7(a). A builder agreed with a homeowner to build a basic frame house for $100,000. The homeowner paid the builder upfront, $100,000. After doing most of the job, the builder repudiated (breached) the contract. Other builders would have charged the homeowner $5,000 to finish the job, but the homeowner found builder X, who needed work and was willing to do it for $4,000. How much can the homeowner recover from the breaching builder? Only $4,000, which is the difference between what it cost the homeowner to complete the home ($104,000, including the $100,000 initially paid to the breaching builder and the $4,000 paid to builder X) and the $100,000 contract price. That $4,000 represents the homeowner’s out of pocket cost to remedy the initial builder’s deficient performance. The builder repaying the homeowner $4,000 will put the homeowner in as good a position as performance by the builder would have. That is, a house built for a cost of $100,000 to the homeowner.

Page 16: CONTRACTS MNEMONICS · 2019. 9. 23. · ©2018 Pieper Bar Review 1 . CONTRACTS MNEMONICS . 1) The ingredients for a valid contract are TACO: T – Definite TERMS, express or implied

©2018 Pieper Bar Review 16

Applying the general formula to this rule, the homeowner’s damages are the value of the promised performance, $100,000, minus the value of what was received ($100,000 – remember, that the homeowner in the end received the house he contracted for) (at this point under the formula we are at $0), plus $4,000 incidental damages paid to builder X as a result of the initial builder’s breach. Under the alternate, modern approach, the amount needed to make the homeowner whole would still be $4,000, consisting of a $4,000 reliance interest. As a result of the breached contract, the homeowner suffered an unanticipated $4,000 cost. 7(b). If instead of paying the builder $100,000 upfront (as described above), the homeowner had initially paid the builder $60,000, the homeowner would owe the builder the remaining $40,000 less the $4,000 cost of completion (i.e. $40,000 minus the $4,000 cost of completion = $36,000). The court would reason that the builder “substantially performed” his obligations (thus making the breach “immaterial”), requiring the homeowner to fulfill her obligation, less the cost of completion.

Page 17: CONTRACTS MNEMONICS · 2019. 9. 23. · ©2018 Pieper Bar Review 1 . CONTRACTS MNEMONICS . 1) The ingredients for a valid contract are TACO: T – Definite TERMS, express or implied

©2018 Pieper Bar Review 17

CONTRACT OFFERS

Effective Acceptance Terminated Revocation Unilateral Contract Offer

When communicated to the offeree.

If the offer cannot be accepted by a promise (exception UCC Sale of Goods contracts), it is accepted when contract performance is fully completed. Under Restatement (Second) Contracts section 45, acceptance occurs when an offeree tenders or begins performance, but performance must be completed as a condition to any recovery.

Passage of stated time or reasonable time, by a counteroffer, an offeree’s rejection of the offer, incompetency of either party, or by an intervening illegally.

Contract permits revocation anytime before performance is fully completed. Restatement (Second) Contracts § 45 makes the offer irrevocable once performance has been tendered or the offeree begins performance.

Bilateral Contract Offer

When communicated to the offeree.

When acceptance of the offer is dispatched (“mailbox rule”), or acceptance is orally communicated to the offeror.

Passage of stated time or reasonable time, by a counteroffer, an offeree’s rejection of the offer, incompetency of either party, or by an intervening illegally.

Communicated to the offeree before acceptance has been dispatched.

Options (Irrevokable Offers)

When communicated to the offeree. Options generally must be supported by consideration (exception GOL section 5–1109 & UCC section 2–205 where a signed writing takes the place of consideration).

Acceptance of the option (called exercising the option) is effective only when the acceptance is received by the offeror.

Passage of the stated date or after a reasonable time, destruction of the subject matter or intervening illegality.

Not revocable by offeror, and not revoked by offeror’s death, or offeree’s rejection or counteroffer. It can be revoked by the offeror if revocation notice reaches offeree before the option does.

Auction Offers (UCC 2–328)

The bid price (offer) is announced by the bidder to the auctioneer.

Auction hammer falls and auctioneer announces “SOLD.”

A higher bid terminated a pending bid.

The bid is orally withdrawn by the bidder before the hammer falls.

Rejection of the Offer

Only when it is received by the offeror.

Implied when offeror receives rejection.

Acceptance of offer is communicated to the offeror before the offeror receives the rejection.

Offeree’s acceptance is mailed, received or is communicated to the offeror before the rejection is received by offeror.

Page 18: CONTRACTS MNEMONICS · 2019. 9. 23. · ©2018 Pieper Bar Review 1 . CONTRACTS MNEMONICS . 1) The ingredients for a valid contract are TACO: T – Definite TERMS, express or implied

©2018 Pieper Bar Review 18

Pre–Existing Duty Rule Summary

Common Law Contracts (Followed by a majority of jurisdictions): A price change had to be supported by new consideration, otherwise the promise to pay the increased price was unenforceable for lack of consideration. The parties’ original price would govern.

UCC Sales Contracts §2-209(1) (Followed by all courts in contracts involving the sale of goods): The agreed price increase is enforceable without any new consideration if based on “good faith” reason for asking for more money to perform the contract. No signed writing is necessary unless the modified total price is $500 or more, which then requires a signed writing. Restatement (Second) Contracts §89 (Minority position): The agreed price increase is enforceable without any new consideration if based on a good faith reason that was not anticipated when the parties originally entered the contract. No signed writing is required to enforce the modification.

Page 19: CONTRACTS MNEMONICS · 2019. 9. 23. · ©2018 Pieper Bar Review 1 . CONTRACTS MNEMONICS . 1) The ingredients for a valid contract are TACO: T – Definite TERMS, express or implied

OFFER AND ACCEPTANCE EXAMPLES

1. On June 1, R mailed to E an offer that stated, “I will sell you Blackacre for

$100,000. This offer is good for 8 days.” (This was not an option because there was nothing in R’s language limiting R’s right to revoke the offer earlier; this language simply sets an expiration date when the offer expires). E received the offer on June 3. Until what day can E accept R’s offer? Until June 11 (eight days after it was received), because E’s power to accept did not exist until the offer was received. However, if the offer was unreasonably delayed, and E, the offeree, was aware of the delay (snowstorm, postal strike, blackout), then the offer could only be accepted within eight days from when it should have arrived. Assume that 5 days after the offer was received, E learned that R had just sold Blackacre to X for $130,000. This communicated fact is an implied revocation of R’s offer. Although R did not expressly notify E, E had knowledge of R’s intent to revoke the offer as implied by R’s sale of the property to X. A revocation by implication must be communicated to the offeree. Assume E timely mailed an acceptance not knowing that R had sold to X. Here there was a valid acceptance (no communication of the implied revocation), however, E’s remedy is limited to a suit for damages for his lost profit ($30,000). E could not seek specific performance if buyer X was a BFP. Assume, unknown to E, Blackacre burned down in the morning and E, faxed an acceptance that afternoon. Here, R’s offer was terminated by operation of law, by destruction of the subject matter of the offer. Knowledge or notice to the offeree is not required because such a contract would be rescinded based on the parties mutual mistake.

OFFER AND ACCEPTANCE Examples

Page 20: CONTRACTS MNEMONICS · 2019. 9. 23. · ©2018 Pieper Bar Review 1 . CONTRACTS MNEMONICS . 1) The ingredients for a valid contract are TACO: T – Definite TERMS, express or implied

2. If after receiving R’s offer, X mailed an acceptance but suddenly changed her mind and Federal Expressed a rejection, there is nevertheless a binding contract at the moment when the acceptance was first dispatched. However, if R received the Fed. Ex. rejection, first, relied on it, and entered a new contract with “Y”, then X is estopped from denying the effectiveness of the rejection because of R’s detrimental reliance.

3. On January 1st, M, a merchant, sent an offer in writing to B to sell 200 widgets at $100 per widget. The offer stated, “This offer is good through March 1.” On March 1, B dropped his acceptance into the mailbox and it was received by M on March 3. Under the “mailbox” rule the acceptance was timely and a binding contract was formed.

A. If M’s January 1 written offer stated “this offer is firm and will not be

revoked until March 1,” then M’s “option” (even though not supported by consideration) is nevertheless good because it was in writing, signed by a merchant. If on March 1, B mailed an acceptance which M received on March 3, is there a binding contract? No, because the acceptance of an option is not effective until it is received. March 3rd is too late because it expired (DIE).

B. If M’s January 1st written offer said “This offer will not be revoked

until 9:00 PM on May 1st,” and on the morning of May 1st, B mailed an acceptance that M received May 3rd. Is there a binding contract?

Yes, because M’s option to sell goods was not supported by

consideration it could not be an option for more than three months. When the three months expired (April 1), it became an ordinary offer containing an expiration date of 9:00 PM on May 1. Thus, under the mailbox rule, when B dropped the acceptance letter into the mailbox on May 1st, before the offer expired, the offer was timely accepted and a contract was formed.

Page 21: CONTRACTS MNEMONICS · 2019. 9. 23. · ©2018 Pieper Bar Review 1 . CONTRACTS MNEMONICS . 1) The ingredients for a valid contract are TACO: T – Definite TERMS, express or implied

©2018 Pieper Bar Review 19

Contracts

Formation, Performance, Breach, and Discharge (50%) What Concepts Do They Expect You to Know? I. Formation of Contracts A. Mutual assent 1. Offer and acceptance 2. Unilateral contracts 3. Bilateral contracts 4. Implied–in–fact contract B. Indefiniteness and absence of terms C. Consideration 1. Bargained-for exchange D. Obligations enforceable without a bargained-

for exchange 1. Reliance 2. Restitution E. Modification of contracts

II. Performance, Breach, and Discharge A. Conditions 1. Express 2. Constructive B. Excuse of conditions C. Breach 1. Material and partial breach 2. anticipatory repudiation D. Obligations of good faith and fair dealing E. Express and implied warranties in sale-of-goods

contracts F. Other performance matters 1. Cure 2. Identification 3. Notice 4. Risk of Loss G. Impossibility, impracticability, and frustration

of purpose H. Discharge of duties 1. Accord and satisfaction 2. Substituted contract 3. Novation 4. Rescission 5. Release

50 % Defenses to

enforceability, Contract Content

and Meaning, Remedies, and

Third-party Rights

50% Formation of

Contracts, Performance, Breach, and Discharge

Breakdown of Contracts on the MBE

Page 22: CONTRACTS MNEMONICS · 2019. 9. 23. · ©2018 Pieper Bar Review 1 . CONTRACTS MNEMONICS . 1) The ingredients for a valid contract are TACO: T – Definite TERMS, express or implied

© 2018 Pieper Bar Review 20

Defenses, Contract Content and Meaning, Remedies, and Third-party Rights (50%) What Concepts Do They Expect You to Know?

I. Defenses to Enforceability A. Incapacity to contract B. Duress and undue influence C. Mistake, misunderstanding D. Fraud, misrepresentation, and nondisclosure E. Illegality, unconscionability, and public policy F. Statute of frauds II. Contract Content and Meaning

A. Parol evidence B. Interpretation C. Omitted and implied terms

III. Remedies A. Expectation interest

a. Direct damages b. Incidental damages c. Consequential damages

B. Causation, certainty, and foreseeability C. Liquidated damages and penalties, and limitation of remedies D. Avoidable consequences and mitigation of damages E. Rescission and reformation F. Specific performance and injunction G. Reliance and restitution interests H. Remedial rights of breaching parties IV. Third-party Rights A. Third–party beneficiaries B. Assignment of rights and delegation of duties

Page 23: CONTRACTS MNEMONICS · 2019. 9. 23. · ©2018 Pieper Bar Review 1 . CONTRACTS MNEMONICS . 1) The ingredients for a valid contract are TACO: T – Definite TERMS, express or implied

© 2018 Pieper Bar Review 21

1. A homeowner advertised his home for sale at an asking price of $100,000. A prospective buyer expressed an interest in buying it at the asking price if the homeowner would paint the exterior. The homeowner agreed, but told the buyer that his attorney was out of town for two weeks and couldn’t draw up the papers immediately. The parties agreed that the homeowner would have the house painted in the meantime.

The next day, the buyer met the painting contractor and selected an unusual shade of purple paint for the house. Ten days later, when the painting contractor was nearly finished painting the house, the buyer received a termination notice from his employer. The buyer immediately informed the homeowner that he would no longer be able to buy the house.

The homeowner approached several realtors regarding the listing of his home, but the realtors informed him that he would need to have the exterior repainted, since the color the buyer had chosen was highly unattractive to most buyers.

If the homeowner sues the buyer to recover his costs in painting the house, is the homeowner likely to prevail?

(A) No, since the homeowner was not

bound under an enforceable contract to have the house painted.

(B) No, since the painting did not constitute an unjust enrichment to the buyer.

(C) Yes, since the buyer breached the contract for the sale of the house.

(D) Yes, under a quasi–contract theory, since the homeowner relied to his detriment on the buyer’s promise to purchase the house.

1. Although the painting of the house was part of the contract for sale which should have been in writing, courts allow recovery on a quasi–contract theory when one party to an oral agreement has performed or partly performed in reliance on the contract. Here, the lead in to the question tells us that the homeowner incurred “costs,” and the reason he incurred those costs was the oral agreement with the buyer. Thus, (A) is an incorrect answer.

While the homeowner cannot compel specific performance of the entire agreement, he can recover the amount he actually expended. Thus, the answer is (D), not (C).

The basis of recovery is not unjust enrichment, so (B) is incorrect.

Page 24: CONTRACTS MNEMONICS · 2019. 9. 23. · ©2018 Pieper Bar Review 1 . CONTRACTS MNEMONICS . 1) The ingredients for a valid contract are TACO: T – Definite TERMS, express or implied

© 2018 Pieper Bar Review 22

2. Which of the following scenarios would warrant a quasi–contractual recovery?

(A) A fierce blizzard surprised a cross

country skier as he skied through the forest. To avoid freezing to death, the skier broke into a nearby chalet, burned the owner’s firewood, and left after the storm subsided. The owner sued the skier for the fair rental value of the chalets use and for the fair market value of the firewood.

(B) A plane crash rendered a passenger unconscious for 24 hours. A physician at the scene of the crash rendered medical assistance to the unconscious passenger and later sued the passenger for the value of his medical services.

(C) A homeowner hired a landscaper to maintain his lawn at 12 Oak Park Drive. The landscaper parked his truck, which has an elaborate sign identifying his lawn service company, in front of the homeowner’s neighbor’s house at 14 Oak Park Drive, and mistakenly mowed the neighbor’s lawn while the neighbor watched from her window. The landscaper sued the neighbor for the fair value of the lawn mowing services.

(D) An elderly uncle told his niece that he would give his house to her if she would live with him and take care of him for one year. At the conclusion of the year, the uncle refused to give his house to the niece, and the niece, unable to obtain specific performance because of the statute of frauds, sued her uncle for the fair market value of the property.

2. A quasi–contract, as distinguished from an implied–in–fact contract, occurs when there is no basis for concluding that the parties by their behavior impliedly entered into a contract. In (A) and (C), the skier and neighbor impliedly agreed by their conduct to pay for the services rendered to him. Thus, each is liable under the theory of an implied–in–fact contract, rather than quasi–contract.

In (B), the passenger has not by his conduct impliedly agreed to pay for the services rendered. Since he was unconscious, he could not have knowingly agreed to or acquiesced to the services provided by the physician. Therefore (B) is the correct answer.

The situation described in (D) also gives rise to quasi–contract (i.e, where an express contract fails because of the statute of frauds, incapacity of one of the parties, illegality, mutual mistake, etc., then in order to avoid the unjust enrichment of one of the parties, a quasi–contract may be imposed). The measure of damages in (D), however, is incorrect; it is not the value of the property, but rather the fair market value of the niece’s services which will be awarded by the court.

Page 25: CONTRACTS MNEMONICS · 2019. 9. 23. · ©2018 Pieper Bar Review 1 . CONTRACTS MNEMONICS . 1) The ingredients for a valid contract are TACO: T – Definite TERMS, express or implied

© 2018 Pieper Bar Review 23

3. A student had just completed his junior year of college and was looking for a summer job. He was approached by a homeowner, who said to him, “I have twenty gallons of paint in my garage. If you will paint my house with it during the next three weeks, I will pay you $1,000.” The student immediately went to the hardware store and purchased three paint brushes for $20. That night, the homeowner telephoned the student and said, “I’m sorry, but my nephew arrived yesterday, and he needs the work badly, so I won’t need you to paint my house.” If the student sues the homeowner for breach of contract, is the student likely to prevail? (A) Yes, because once an offeree

commences work pursuant to a unilateral contract offer, the offer becomes irrevocable.

(B) Yes, because the homeowner induced the student to purchase the paint brushes, and promissory estoppel would effectively convert the homeowner’s offer into an option contract.

(C) No, because an offeror may revoke a unilateral contract at any time before it is accepted by completion of the act requested.

(D) No, because the homeowner revoked his offer before it was accepted.

3. An offer for a unilateral contract becomes irrevocable when the offeree tenders performance or commences the work requested, but not when there is only mere preparation to perform that work. The purchase of paint brushes is only preparation and therefore the homeowner had the right to revoke the offer at the time he did. (D) is the correct answer.

(C) is an overly broad statement of the homeowner’s right to revoke. He could not revoke at any time prior to completion, but only before work on the requested project commenced. The performing party has a cause of action for restitution in quasi contract, but no claim for breach of contract.

(B) is incorrect because the reliance in this case, the purchase of $20 worth of paint brushes, was not of such substantial character that the doctrine of promissory estoppel under §87(2) of the Restatement (Second) of Contracts would convert the offer into an option contract. The courts will only award a remedy on the grounds of promissory estoppel where the reliance was substantial (e.g., a substantial expense or forgoing substantial alternatives) and foreseeable. Here the economic injury suffered is not of unconscionable proportion and enforcement of the contract would not be “necessary to avoid injustice.” This answer choice may be tempting since it provides an equitable solution to the student, but it is incorrect (the bar examiners like to provide answer choices that suggest a fair outcome on equitable grounds to throw you off). (A) is incorrect because the student had not begun the work. Here, homeowner revoked the offer before the student either tendered performance or commenced the paint job. The student had merely prepared to start the paint job. Joseph M. Perillo, Contracts § 2.22 at 99-100 (7th ed. 2014).

Page 26: CONTRACTS MNEMONICS · 2019. 9. 23. · ©2018 Pieper Bar Review 1 . CONTRACTS MNEMONICS . 1) The ingredients for a valid contract are TACO: T – Definite TERMS, express or implied

© 2018 Pieper Bar Review 24

4. A store owner received a written offer from a clothing wholesaler to purchase 600 Oxford style, white, cotton shirts for $2 per shirt, totaling $1,200. The wholesale price for similar shirts is typically $20 per shirt. The store owner thought that the wholesaler must have lost his mind, but, nevertheless, agreed to the offer. The parties each signed a purchase order which called for delivery of the shirts to the owner’s place of business and payment of the purchase price on July 1. Two days later, the wholesaler called the owner and told him that he had made a mistake because he had read from the wrong line of his price list. He said that unless the owner paid him $20 per shirt, the price of the shirt on the correct line of the price list, or a total price of $12,000, he would not deliver the shirts. The owner refused to pay anything more than $1,200. When the wholesaler failed to deliver the shirts on July 1, the owner purchased 300 comparable shirts for $20 per shirt and brought suit against the wholesaler for damages resulting from the wholesaler’s breach of contract.

Which of the following will the court conclude?

(A) The wholesaler will prevail

because he made a unilateral mistake.

(B) The wholesaler will prevail because of the doctrine of mutual mistake.

(C) The wholesaler will prevail since it was obvious to the owner on May 1 that the wholesaler made a blatant mistake in quoting the price of $2 per shirt.

(D) The owner will prevail because he accepted the offer.

4. A contract can be rescinded for unilateral mistake such as this one only when the unilateral mistake was so obvious that the other party must have known that the first party made a mistake.

This question clearly indicates that a unilateral mistake was made by the wholesaler, but that fact alone is not sufficient reason to permit him to rescind the contract. Because (C) is more precise, (A) is not the best answer.

(B) is clearly incorrect because there was no mutual mistake by both parties. There is only a unilateral mistake.

(C) is the correct answer. There is a substantial disparity between the contract price of $2 per shirt and the market value of the shirts at wholesale, which seems to be $20 a shirt. If the buyer, because of the disparity in price, was aware or should have been aware that the seller had made a mistake, the seller will be able to rescind the contract and, therefore, will prevail.

(D) is incorrect because the owner will not prevail since he was aware of the mistake.

Page 27: CONTRACTS MNEMONICS · 2019. 9. 23. · ©2018 Pieper Bar Review 1 . CONTRACTS MNEMONICS . 1) The ingredients for a valid contract are TACO: T – Definite TERMS, express or implied

© 2018 Pieper Bar Review 25

5. On March 18, a department store published an ad in the local newspaper that stated:

“Now that winter is behind us, we will be selling our inventory of forty mink coats (which have a retail price of $2,000 per coat) for $600 each, at 8:30 a.m., Saturday, March 20. One coat per person. First Come First Served.”

The owner of a nearby fur store decided to purchase one of the coats to hold in her inventory until next fall. She was the first customer in line when the department store opened its doors on March 20. She tendered $600 in cash and asked to purchase one of the advertised mink coats. The manager of the fur department recognized her as the owner of a competing fur store and refused to sell her a coat.

The owner of the fur store brought an action against the department store seeking a judgment that it deliver a coat to her in exchange for $600, or, in the alternative, for damages resulting from the department store’s failure to sell her a coat.

In her action against the department store, will the owner of the fur store prevail?

(A) Yes, on either remedy, because her

tender of the $600 was a valid acceptance of the department store’s offer.

(B) Yes, because the department store breached a valid contract, but the owner of the fur store will only be able to collect money damages.

(C) No, on both remedies, because an advertisement to the general public is only an invitation to make an offer, and when the owner of the fur store made the offer, the department store properly refused to accept it.

(D) No, on both remedies, because by placing the “one coat per person” restriction in the ad, the department store was reserving its right to refuse to sell coats to competitors.

5. An advertisement indicating a present intention to sell to an individual who meets specific criteria (for example, being one of the first customers at the store on the day of the sale) constitutes an offer. The owner of the fur store properly accepted this offer by arriving at the store and tendering $600 for the coat. She will not obtain specific performance, however, because the coat was not unique. There were forty in stock at the store, and damages will be a sufficient remedy. She will be entitled to recover the difference between the fair market value of the coat and $600. Therefore, (B) is correct and (A) is incorrect.

(C) is incorrect. Although most advertisements do not constitute offers, the criteria of this particular advertisement were specific enough to constitute an offer.

The advertisement was not to the general public but only to the first forty customers who accepted. Customers reasonably understand that they have the power to accept when an advertisement identifies the quantity to be sold. Murray on Contracts §35 at 79 (5th ed. 2011).

(D) is incorrect because the “one coat per person” limitation does not reasonably indicate to a purchaser that there is any limitation on who may purchase a coat.

Page 28: CONTRACTS MNEMONICS · 2019. 9. 23. · ©2018 Pieper Bar Review 1 . CONTRACTS MNEMONICS . 1) The ingredients for a valid contract are TACO: T – Definite TERMS, express or implied

© 2018 Pieper Bar Review 26

6. A series of seven arsons occurred at a city’s homeless shelter. On January 15, the city council adopted a resolution that stated:

The city will pay $15,000 for the arrest and conviction of anyone found guilty of the seven arsons at the city’s homeless shelter.

The city council proceedings were telecast live over the city’s local access cable channel. No other publicity was disseminated by the city.

On January 25, the city council passed a resolution repealing its reward offer because the city was facing budget constraints. The January 25 proceedings were not telecast over the local access cable channel because the channel was televising the city’s high school basketball tournament that night. The city council documented the resolution in the minutes of its proceedings, and the minutes were published in the legal notices in the back pages of the local paper.

On February 1, a bar patron overheard a conversation implicating a disturbed electrician in the arsons. Since he knew of the reward, but had not heard of its revocation, the patron relayed the information to the police. The police lawfully arrested the electrician who confessed and was subsequently convicted of the arsons. The bar patron sought to recover the reward money, but the city refused.

If the bar patron sues the city to recover the reward, will he prevail?

(A) Yes, because he knew of the January 15

resolution at the time he talked to the police and had no knowledge of the January 25 resolution repealing the city’s offer.

(B) Yes, because he had no knowledge of the January 25 resolution repealing the city’s offer and his knowledge of the January 15 resolution is irrelevant.

(C) No, because the January 25 resolution effectively revoked the offer.

(D) No, because the offer was not accepted within a reasonable time.

6. The January 15 resolution was a unilateral contract offer which could be accepted by anyone with knowledge of that offer. It was a specific offer for a specific crime as contrasted with a standing offer to reward for any particular crime, e.g., “$10,000 to anyone who provides information leading to the capture of any killer of a police officer.” This latter type offer can be claimed even if the offeree was unaware of it.

Restatement (Second) Contracts § 23 provides that “[s]tanding offers of rewards made by governmental bodies [are] intended to create a climate in which people do certain acts in the hope of earning unknown rewards.” Illustration 3 under § 23 explains, “A city ordinance provides a standing reward of [$50,000] will be paid for information leading to the arrest and conviction of anyone guilty of [any] arson within the city limits. A furnishes such information. A is entitled to the reward whether or not he knew of the reward or was motivated by hope of the reward.” Id. Here, the January 25 resolution revoking the earlier offer was not publicized in such a way as to revoke the offer except as to those who had actual knowledge of the revocation. (A), which incorporates both of these concepts, is the correct answer.

(B) is incorrect because it suggests that the bar patron could have claimed the reward even if he had not known of the January 15 reward offer. A person who acts without knowledge of an offer is generally incapable of enforcing a contract. The only exception is standing offers of reward made by the government.

(C) is incorrect because the January 25 council proceeding was not televised. Since the purported revocation did not receive publicity comparable to the January 15 offer, it would be ineffective. Murray on Contracts § 43 at 121 (5th ed. 2011); Joseph M. Perillo on Contracts § 220(d) at 87–88 (7th ed. 2014).

(D) is incorrect because the offer was accepted within two weeks from the time it was made, which is a reasonable time for a reward offer to remain outstanding.

Page 29: CONTRACTS MNEMONICS · 2019. 9. 23. · ©2018 Pieper Bar Review 1 . CONTRACTS MNEMONICS . 1) The ingredients for a valid contract are TACO: T – Definite TERMS, express or implied

© 2018 Pieper Bar Review 27

7. Prior to the start of an auction, each bidder and the auctioneer signed documents sufficient to satisfy the statute of frauds should a contract be formed at the auction, and the auctioneer announced that the auction would be “without reserve.” The auctioneer then began to auction a valuable Picasso painting. The first bidder bid $100,000 for the painting. A second bidder immediately thereafter bid $125,000. One minute later, the second bidder withdrew his bid. No further bids were made and nothing relevant was said during the next five minutes. At that time, the auctioneer announced that the auction sale was terminated.

Which of the following is a correct statement of the parties’ legal rights and duties?

(A) There is a binding contract

between the auctioneer and the second bidder for $125,000.

(B) There is a binding contract between the auctioneer and the first bidder for $100,000, because the second bidder’s withdrawal revived the prior bid.

(C) The auctioneer is not contractually bound to sell the painting, because no bids were made within a reasonable time of the second bidder’s revocation.

(D) The auctioneer is not contractually bound to sell the painting, because he could withdraw the painting from auction at any time before his hammer fell.

7. Auctions of personal property are governed by UCC §2–328.

(A) is incorrect because, under UCC §2–328(3), a bidder may retract his bid at any time before the auctioneer’s hammer falls.

(B) is incorrect because, under UCC §2–328(3), the retraction of a bid does not revive a prior bid.

If the auction sale is “without reserve,” the auctioneer cannot withdraw the article from auction. (D) is therefore incorrect.

However, if there are no bids within a reasonable time, an auctioneer can terminate an auction that is “without reserve.” Since this occurred, (C) is correct.

Page 30: CONTRACTS MNEMONICS · 2019. 9. 23. · ©2018 Pieper Bar Review 1 . CONTRACTS MNEMONICS . 1) The ingredients for a valid contract are TACO: T – Definite TERMS, express or implied

© 2018 Pieper Bar Review 28

8. On June 1, a property seller mailed a signed letter to a buyer which stated:

“I will sell you my house at 23 Garden Lane in Louisville for $150,000. I want a deposit of $15,000 and will close at the registry of deeds on August 1st. Please reply by June 15.”

On June 3, the buyer wrote back:

“Thank you. Would you consider selling for $145,000?”

On June 5, the seller wrote to the buyer:

“I have received your letter of June 3. I reject your offer to buy my house for $145,000.”

On June 7, the buyer wrote to the seller:

“I have received your letter of June 5, and accept the offer contained in your letter of June 1. I am enclosing herewith my check in the amount of $15,000 as the deposit you requested. Please deliver a deed to the property conveying marketable title at the closing on August 1, at which time I will pay to you the balance of the purchase price.”

On June 9, the seller wrote to the buyer:

“I am returning herewith your check for $15,000. I am not contractually obligated to sell you my house at 23 Garden Lane, Louisville.”

The buyer appeared at the Louisville registry of deeds on August 1, prepared to tender the entire $150,000 purchase price, but the seller did not appear. The buyer brought an action against the seller for specific performance of the contract.

Will the buyer likely prevail?:

(A) No, because his June 3 letter was a

counter–offer. (B) No, because the seller’s June 5 letter

revoked his June 1 offer. (C) No, because the buyer’s June 7 letter was

not an effective acceptance because it contained additional terms.

(D) Yes, the buyer will prevail.

8. (A) is incorrect because the June 3 letter would best be characterized as an inquiry. An inquiry does not constitute a counter–offer and therefore is not a rejection of the original offer.

(B) is incorrect because the June 5 letter was a reply to the inquiry of June 3, and did not by its terms revoke the June 1 offer. This choice incorrectly characterizes the June 3 letter as a counter–offer, when it was really only an inquiry.

(C) is incorrect because the additional terms set forth in the June 7 letter were terms that were implied in the original offer (namely, that the seller convey marketable title). As we will cover in further detail in real property, if a contract for the sale of real property makes no mention of the quality of title to be conveyed, it is implied in the contract (not in the deed) that the seller will tender marketable title at closing.

(D) therefore is correct because the buyer accepted the seller’s June 1 offer though his June 7 letter.

Page 31: CONTRACTS MNEMONICS · 2019. 9. 23. · ©2018 Pieper Bar Review 1 . CONTRACTS MNEMONICS . 1) The ingredients for a valid contract are TACO: T – Definite TERMS, express or implied

© 2018 Pieper Bar Review 29

9. A university advertised for bids for a new ice hockey arena. The bids were to be submitted June 1 and opened on June 15, at which time the contract would be awarded. One of the major items included in the bid was a large compressor.

A general contractor planned to bid on the project and asked several compressor dealers to bid on the required compressor. He indicated to the dealers that if he won the contract, he would enter into a contract to buy the compressor from the lowest bidder on June 20, with delivery to take place on August 20. On May 15, a large compressor dealer submitted a written offer to sell to the contractor the required compressor for $75,000. This was the lowest offer received by the contractor. On May 20, the contractor called the dealer, stated his intent to bid on the university ice hockey arena, and asked if the dealer would hold his offer open until June 21. The dealer said yes. The contractor then submitted a $5,000,000 bid to construct the ice hockey arena.

The university awarded the contract to the contractor, who was the lowest bidder, on June 15. On June 16, the dealer learned that the contractor received the contract, immediately called the contractor, and revoked the offer to sell the compressor for $75,000. On June 17, the contractor wrote to the dealer and accepted the offer to sell the compressor for $75,000. When the dealer did not deliver the compressor on August 20, the contractor purchased the compressor elsewhere for $90,000. In an action by the contractor against the dealer to recover the $15,000 difference between the purchase price and the amount of the dealer’s offer, will the contractor prevail?

(A) Yes, because the dealer’s offer was a

“firm” offer under the Uniform Commercial Code and could not be revoked until June 21.

(B) Yes, because the dealer’s agreement to keep the offer open until June 21 was enforceable since the dealer knew that the offer would induce substantial reliance on the part of the contractor.

(C) Yes, because a written offer must be revoked by a writing.

(D) No, because the dealer validly revoked his offer before the contractor accepted it.

~ ~ ~ ~ ~ 9. (B) is correct. The offer made by the

dealer to the contractor is governed by the rules of promissory estoppel set forth in §87(2) of the Restatement (Second) of Contracts. When the dealer made the offer, he knew it would induce substantial reliance by the contractor in bidding on the skating rink contract. Therefore, an option contract was created and the option was properly exercised by the contractor.

(A) is wrong because the promise to keep an offer open under UCC §2–205 must be in writing. Here, the dealer’s promise to keep the offer open was oral.

(C) is wrong because the offer, if it was revocable at all, could have been revoked by a telephone call.

(D) is incorrect because the offer was not revocable before June 21 for the reasons set forth in (B).

Page 32: CONTRACTS MNEMONICS · 2019. 9. 23. · ©2018 Pieper Bar Review 1 . CONTRACTS MNEMONICS . 1) The ingredients for a valid contract are TACO: T – Definite TERMS, express or implied

© 2018 Pieper Bar Review 30

10. On March 16, an artist mailed to a patron an offer to paint the patron’s portrait for $2,500. On April 1, the patron mailed back his acceptance, but conditioned the acceptance upon the painter framing the portrait in a frame worth at least $500.

On April 2, the patron found in his attic a frame perfect for the portrait. That same day, the patron faxed to the artist an acceptance of the artist’s offer and advised the artist that he had a suitable frame in which to put the painting.

On April 3, the artist received the patron’s April 1 correspondence.

What is the status of the parties relationship on April 4?

(A) There is no contract to paint the

portrait for $2,500 because of the “mailbox rule.”

(B) There is no contract to paint the

portrait because the patron’s counter–offer rescinded the artist’s offer.

(C) There is a contract to paint the

portrait for $2,500, and for the artist to supply a $500 frame.

(D) There is a contract to paint the

portrait for $2,500 but no obligation for the artist to supply a $500 frame.

10. A rejection terminates the offeree’s power of acceptance. A counter–offer (a new proposal), because it is an implied rejection of the offer, has the same effect as an express rejection. Restatement (Second) Contracts § 39. Thus, (C) is not correct because the artist’s obligation to obtain the frame was a counter–offer that the artist never agreed to.

(B) otherwise would be correct except for the fact that a rejection (the April 1 letter) is effective only when received (April 3) by the offeror (the artist). (B) is not correct because, before the artist received the counter–offer (rejection), he received the April 2 acceptance by fax. Thus, there was a binding contract as of April 2 and (D) is correct.

(A) is incorrect because the mailbox rule states that an acceptance is effective when dispatched and put out of the offeree’s control (dropped into the mailbox), and it applies to an “acceptance” and not a rejection of the offer (a counter–offer). A rejection is not effective when dropped into the mailbox, but rather only when received by the offeror, here the artist.

Page 33: CONTRACTS MNEMONICS · 2019. 9. 23. · ©2018 Pieper Bar Review 1 . CONTRACTS MNEMONICS . 1) The ingredients for a valid contract are TACO: T – Definite TERMS, express or implied

© 2018 Pieper Bar Review 31

11. The owner of a chain of fast food restaurants entered into a written contract with a manager to operate one of her restaurants for two years at a salary of $52,000 per year, payable at the rate of $1,000 per week. After the manager had worked at the restaurant for 16 months, he suggested to the owner a way of recycling the fat used to cook french fries which would result in significant savings in both purchase and disposal costs for the entire restaurant chain. The suggestion was implemented immediately and was very successful.

Thereafter, the owner told him that she was so pleased with his fat recycling suggestion, that she was raising his pay to $2,500 per week for the remainder of the contract.

When the manager’s next paycheck arrived, it was only for $1,000. He approached the owner and she told him that, after talking with her accountant, she had decided not to raise his pay, but would continue to honor the conditions of the original contract.

If the manager brings an action against the owner to secure the additional $1,500 per week, will he prevail?

(A) Yes, because he conferred a material

benefit on the owner. (B) Yes, because his promise to manage

the restaurant for the remainder of the contract term would make the owner’s promise enforceable.

(C) No, because past consideration would not support the owner’s promise.

(D) No, because a contract for more than a year in duration must be in writing to be enforceable.

11. The suggestion to recycle fat was not part of any bargain between the manager and the owner, and neither past nor “moral” consideration is sufficient to support the owner’s promise to pay the manager more money for the job he was already contractually obligated to perform for $1,000 per week. Since this contract is governed by common law rules, new or additional consideration is necessary to support a modification. Therefore, (C) is correct.

(A) is incorrect because the conferring of a material benefit, if not bargained for, does not support a contract. The benefit conferred by the manager was not “bargained for” as part of the owner’s promise since it already had been performed by the manager.

(B) is incorrect because the manager is already contractually bound to manage the restaurant for the remainder of the term, and his promise to live up to his existing contract cannot furnish consideration for a modification of that contract.

(D) is incorrect because, at the time the owner promised to increase the manager’s salary, the contract had less than one year to run and therefore the promise would not be unenforceable because of the statute of frauds.

Page 34: CONTRACTS MNEMONICS · 2019. 9. 23. · ©2018 Pieper Bar Review 1 . CONTRACTS MNEMONICS . 1) The ingredients for a valid contract are TACO: T – Definite TERMS, express or implied

© 2018 Pieper Bar Review 32

12. A talented sailor was tied for the lead in a prestigious weekly race series, with one week remaining before the final race of the season. Unfortunately, his boat began to leak and required emergency repairs if the sailor expected to compete in the final race. A local carpenter told the sailor that because his regular boss did not have any work for him for the coming week, he would repair the boat for $1,200 paid in advance. The sailor paid the $1,200 on the condition that the repairs be completed so that the boat could sail the following weekend, and the carpenter began the repairs.

On Thursday morning the carpenter informed the sailor that his regular boss needed him for an emergency job on Thursday and Friday. His boss had offered to pay him $500 per day to work for him, so the carpenter told the sailor he would not finish the boat by Saturday unless the sailor paid him an additional $300 on Saturday and agreed not to sue the carpenter for his anticipatory breach of contract. Because the sailor was anxious to win the race series, he agreed to the carpenter’s proposal. The carpenter finished the boat by Saturday morning. When the sailor took possession of the boat, he refused to pay the carpenter the additional $300 promised on Thursday.

If the carpenter brings an action against the sailor in a common law jurisdiction, will he be able to recover the $300?

(A) Yes, because the emergency excused the

carpenter’s original performance deadline, and his promise to complete the job despite the emergency was valid consideration for the second agreement.

(B) Yes, because the sailor’s promise not to sue for the anticipatory breach of contract was a bargained–for exchange for the carpenter’s promise to complete the job on time.

(C) No, because of the preexisting duty rule. (D) No, because the carpenter’s conduct was

unconscionable.

12. The carpenter was under a legal duty to complete the repairs by Saturday. Since the facts state that the action was brought in a common law jurisdiction (and not in a jurisdiction recognizing the Restatement (Second) of Contracts) and the contract was not a contract for the sale of goods governed by the Uniform Commercial Code, consideration is required for a modification of the contract. The carpenter is doing exactly what he originally promised to do, and, therefore, there is no consideration to support the promise to pay the additional $300. Therefore, (C) is correct.

(D) is an attractive choice but wrong, because the conduct of the carpenter is not sufficiently unconscionable. He originally agreed to do the work at a low price because he was out of work, and he started the job in good faith. It was only because his regular boss called him and offered him more money that he asked for additional compensation for the sailor’s job. The preexisting duty rule clearly applies here, making (C) the better answer.

(A) is incorrect because of the preexisting duty rule. The carpenter’s promise is to do exactly what he is already legally obligated to do.

(B) is incorrect because the sailor is not bargaining with the carpenter to give up his lawsuit in exchange for agreeing to pay the carpenter an additional $300. Both of these promises are to the sailor’s detriment, and there is no new promise to the carpenter’s detriment

Page 35: CONTRACTS MNEMONICS · 2019. 9. 23. · ©2018 Pieper Bar Review 1 . CONTRACTS MNEMONICS . 1) The ingredients for a valid contract are TACO: T – Definite TERMS, express or implied

© 2018 Pieper Bar Review 33

13. A man applied for and obtained a $100,000 life insurance policy. The policy contained the following language:

“This policy shall not insure against a death which occurs when the insured is scuba diving, unless he is diving with a certified P.A.D.I. instructor.”

Two months after the policy was issued, the man was scuba diving in the company of the senior dive instructor. The instructor was certified by S.S.I., a training organization similar to P.A.D.I., and had 30 years of scuba diving experience. While the man and the instructor were diving, a great white shark appeared and attacked the man, killing him instantly. Prior to this attack, no great white sharks had been spotted in or near those waters for more than one hundred years.

The beneficiary of the policy bought an action to collect on the policy, but the insurance company refused to pay.

What is the insurance company’s best defense?

(A) The death would not have occurred

if the insured had the benefit of a certified P.A.D.I. instructor’s experience during the fatal dive.

(B) Scuba diving is a hazardous activity properly excluded from the policy.

(C) The insured failed to comply with a contractual condition.

(D) The purpose of the policy exclusion was to protect the company against the type of scuba diving fatality which occurred.

13. (C) is the correct answer. The language quoted from the policy is a condition precedent to performance, which must be satisfied before there can be a recovery under the contract.

(B) is incorrect because scuba diving is not excluded from the policy. Only diving without a certified P.A.D.I. instructor will result in application of the exclusion.

(A) and (D) are also incorrect. Whether the statements are true or whether the death would have resulted under different conditions is irrelevant. The specific language of the condition is controlling.

Thus, a condition precedent to the performance of this contract (payment of money) was that any drowning death by scuba diving be in the presence of P.A.D.I. instructor. Joseph M. Perillo, Contracts §11.5 at 382 (7th ed. 2014).

Page 36: CONTRACTS MNEMONICS · 2019. 9. 23. · ©2018 Pieper Bar Review 1 . CONTRACTS MNEMONICS . 1) The ingredients for a valid contract are TACO: T – Definite TERMS, express or implied

© 2018 Pieper Bar Review 34

14. A homeowner and a painter entered into a contract to paint the homeowner’s house for $700 prior to a wedding that was being held in the homeowner’s backyard. The contract specifically stated that payment was contingent upon completion by the wedding date.

The next day, the painter was offered a more lucrative job, so the painter told a handyman that he was the owner of the house, and that he needed the house painted by the wedding day for $500. The painter paid the handyman, who began work on the job, but failed to complete the job on time.

If the homeowner brings an action against the painter and the handyman, from whom can he recover damages?

(A) The painter only. (B) The handyman only. (C) Both the painter and the

handyman. (D) Either the painter or the handyman,

at the homeowner’s election, but not both.

14. The homeowner has a valid cause of action against the painter because the painter was a party to the original contract and was not discharged from his obligations when he delegated the work.

On the other hand, the handyman, who is really a subcontractor, is not liable to the homeowner because the two are not in privity. Indeed, the painter told the handyman that the painter was the owner of the house, so the handyman had no reason to know of the actual homeowner’s interest in the completion of the paint job. Moreover, because the painter did not relay the urgency of painting the house, it is unlikely that the painter can recover from the handyman as time was not made “of the essence” in the contract between the painter and the handyman.

(A) is therefore correct.

Page 37: CONTRACTS MNEMONICS · 2019. 9. 23. · ©2018 Pieper Bar Review 1 . CONTRACTS MNEMONICS . 1) The ingredients for a valid contract are TACO: T – Definite TERMS, express or implied

© 2018 Pieper Bar Review 35

15. A patient owed her physician $25,000 for professional services. The physician orally assigned this claim to her adult daughter as a wedding gift. Shortly thereafter, after suffering sudden, severe losses in the stock market, the doctor assigned the same claim to her stockbroker in a signed writing, in partial satisfaction of advances legally made by the stockbroker. Subsequently, the patient, without knowledge of either assignment, paid the physician the $25,000 then due, which the physician promptly lost at a horse track, although she remains solvent.

Assuming that Article 9 of the Uniform Commercial Code does NOT apply to either of the assignments in this situation, which of the following is a correct statement of the parties’ rights and liabilities?

(A) As the assignee prior in time, the

physician’s daughter can recover $25,000 from the patient, who acted at her peril in paying the physician.

(B) As the sole assignee for value, the stockbroker can recover $25,000 from the patient, who acted at her peril in paying the physician.

(C) Neither the physician’s daughter nor the stockbroker can recover from the patient, but the physician’s daughter, though not the stockbroker, can recover $25,000 from the physician.

(D) Neither the daughter nor the stockbroker can recover from the patient, but the stockbroker, though not the physician’s daughter, can recover $25,000 from the physician.

15. Generally, an assignment terminates the assignor’s right to collect the contract benefit (here, the account receivable), but the assignee, absent a filing of a UCC Article 9 financing statement, must immediately notify the obligor owing the money (here, the patient) of the assignment. Since neither assignee notified the patient, she satisfied her debt by paying the physician. Choice (D) is the correct answer.

Choice (A) is not the correct answer because, even though the “first–in–time” rule generally prevails, it does not apply when the first assignee was gratuitous. A gratuitous assignment is revocable until the gratuitous assignee gets paid, recovers a judgment against the person owing the money, or enters a new agreement with the obligor. Thus, the physician’s daughter has no claim against either the physician or the patient since her rights were revoked by (1) the reassignment of the same right, as well as (2) the payment of the debt to the physician. Choices (A) and (C) are incorrect.

Although past consideration makes the assignment to the stockbroker non–gratuitous, because the stockbroker did not notify the patient, the patient’s payment of the debt to the physician (the assignor) extinguished the patient’s obligation. Thus, choice (B) is incorrect.

Choice (D) is the best choice. Even though the physician’s daughter’s gratuitous assignment right was extinguished by the physician’s reassignment of the same right to the stockbroker, and the stockbroker’s right against the patient was extinguished by his failure to give notice to the patient prior to her making payment in full to the physician, none of this extinguished the original $25,000 claim the stockbroker had against the physician which was supported by consideration. Thus, the stockbroker has a claim against the physician.

Page 38: CONTRACTS MNEMONICS · 2019. 9. 23. · ©2018 Pieper Bar Review 1 . CONTRACTS MNEMONICS . 1) The ingredients for a valid contract are TACO: T – Definite TERMS, express or implied

© 2018 Pieper Bar Review 36

16. A farmer entered into a contract to sell her entire tobacco crop to a buyer for $100,000, to be paid by the buyer upon delivery. Thereafter, the farmer purchased a race horse from a breeder, and, in writing, assigned her right to payment from the buyer to the breeder as payment for the horse. The race horse was diseased when the breeder delivered it to the farmer and it died the next day.

If delivery of the diseased race horse constituted a breach of warranty of merchantability by the breeder, what is the farmer’s right vis–a–vis the assigned rights to the tobacco crop?

(A) The assignment to the breeder from

the farmer would be revoked automatically.

(B) The farmer could revoke the assignment to the breeder by giving him notice.

(C) The farmer could only revoke the assignment through an action for rescission.

(D) The farmer could not revoke the assignment by any procedure.

16. Choice (C) is the correct answer. An assignment for consideration is in effect a transfer of title to, or ownership of, a contract right. The transferee, or assignee, takes title to this contract right to collect money. In order to revoke the assignment, an action for rescission would have to be commenced. The farmer does have a right to rescind here, because she had a right to reject the race horse, however, it must be accomplished through court action.

The farmer cannot revoke the assignment by any other method. Therefore, choices (A) and (B) are incorrect.

Choice (D) is incorrect because the farmer may revoke the assignment.

Page 39: CONTRACTS MNEMONICS · 2019. 9. 23. · ©2018 Pieper Bar Review 1 . CONTRACTS MNEMONICS . 1) The ingredients for a valid contract are TACO: T – Definite TERMS, express or implied

© 2018 Pieper Bar Review 37

17. A real estate investor entered into an agreement for a builder to construct a six bedroom summer home and a three bedroom summer home that the investor wanted to rent out for the summer months. The investor was to pay for each house when each was completed. The builder agreed to have the six bedroom home completed by March 15, and the three bedroom home completed by March 30. A pipe burst in the three bedroom home during construction and destroyed all of the rooms but the den. The six bedroom home was not completed until June 15, which was too late to take advantage of the summer rental market.

The real estate investor refused to pay the builder for the six bedroom house when the bill was tendered on June 25.

Will the builder succeed in its breach of contract action against the real estate investor?

(A) No, because the law implies that

the time set forth in the contract is of the essence.

(B) Yes, because the contract did not specifically provide that time was of the essence.

(C) Yes, because the flood in the three–bedroom home would extend the time for completion of the six–bedroom home.

(D) Yes, under the common law, but not if the Uniform Commercial Code was enacted by the jurisdiction.

17. In contracts other than UCC contracts, time usually is not of the essence unless the parties make it an express condition of the contract or the circumstances indicate that the parties intended time to be of the essence.

(A) is not correct. “A party [generally]

need not perform on the precise day stated in the contract unless time is made of the essence.” Joseph M. Perillo, Contracts § 11.18 at 397 (7th ed. 2014). Note, however, although usually construction contracts are not made time of the essence, they may be of the essence even though it is not expressly stated. It depends on the parties’ intent, and the circumstances. Id. at n.24. Since the circumstances here did not indicate that time was of the essence, and the contract did not specifically provide that time was of the essence, (B) is the best answer.

Since the two contracts were divisible, the flood in the three bedroom house would extend its completion date, but it would not affect the completion date for the six bedroom house. Thus, (C) is an incorrect answer.

The UCC does not apply to building contracts. Thus, (D) is an incorrect answer.

Page 40: CONTRACTS MNEMONICS · 2019. 9. 23. · ©2018 Pieper Bar Review 1 . CONTRACTS MNEMONICS . 1) The ingredients for a valid contract are TACO: T – Definite TERMS, express or implied

© 2018 Pieper Bar Review 38

18. The owner of a female horse that won horse racing’s most prestigious event was unable to breed the horse when its racing career ended. The owner entered into a written contract to sell the horse for $50,000 to a veterinarian, who truthfully described himself as an avid horse racing fan. $50,000 was a generous price for a sterile thoroughbred, but far less than what the owner could have obtained if the horse had been capable of reproducing.

Using his expertise in breeding animals that had previously been incapable of conceiving, the veterinarian extracted eggs from the horse and fertilized them in vitro. When the owner learned that the horse was pregnant, he brought an action to rescind the sale.

Will the owner prevail?

(A) No, because he has no basis to rescind this valid contract.

(B) No, because of mutual mistake. (C) Yes, because the veterinarian failed

to disclose that he specialized in the breeding of previously infertile animals.

(D) Yes, because his ignorance of the fact that the horse was capable of being bred successfully constitutes unilateral mistake.

18. In this case, the veterinarian saw an opportunity to use his unique skills to create great value from a retired race horse. He was not mistaken concerning the ability of the horse to produce offspring. Therefore, there was no mutual mistake and (B) is incorrect.

While the veterinarian probably knew that the owner thought that the horse would never have offspring, it was not a proven fact at the time of the sale that it could not reproduce. The facts do not indicate that it was pregnant at the time of the sale. A mistaken belief about possible future events does not form the basis for rescission for unilateral mistake. This doctrine is reserved for an obvious mistake of fact known at the time the contract was completed. (D) is therefore incorrect.

The owner and the veterinarian bargained at arms length. Although the owner did not disclose his plans, he made no misrepresentation. He is not, in fact, obligated to make full disclosure of his plans. Therefore, the owner may not rescind the contract for fraud and (C) is incorrect.

(A) is correct because there is no basis to rescind this contract. Note, if the horse was pregnant at the time of the sale, then the contract could be rescinded based on the parties mutual mistake.

Page 41: CONTRACTS MNEMONICS · 2019. 9. 23. · ©2018 Pieper Bar Review 1 . CONTRACTS MNEMONICS . 1) The ingredients for a valid contract are TACO: T – Definite TERMS, express or implied

© 2018 Pieper Bar Review 39

19. An elderly man whose health was declining relied on his nephew to manage his finances. The nephew would fill out checks for the elderly man and then the elderly man would sign them.

The nephew suggested that the elderly man purchase a boat, and after many discussions, convinced the elderly man to purchase the nephew’s boat for $20,000, an amount substantially more than the boat was worth.

Does the elderly man have a valid basis to vacate the transaction?

(A) Yes, based on misrepresentation. (B) Yes, based on duress. (C) Yes, based on undue influence. (D) Yes, based on mistake.

19. Choice (A) is incorrect because the facts do not indicate that the nephew induced the elderly man to enter the transaction based on any misrepresentation or that the elderly man relied on (was deceived by) any misrepresentation.

Mistake as to value generally is not a basis to rescind a contract. Thus, the best answer is not choice (D).

The elderly man cannot successfully argue duress because there were no economic or other threats brought to bear on him. Recognize that the theory of duress only permits a party to avoid a transaction where a wrongful act or threat overcomes the free will of that party and results in an oppressive or abusive contract. Here, the elderly man could have easily elected not to purchase the boat. Thus, choice (B) is not the best choice.

Undue influence speaks of “unfair” persuasion by one party misusing a position of trust and confidence, or by using a dominant psychological position in an unfair manner. Unfairness may be established by the excessive or inadequate consideration involved. Choice (C) is the best choice.

Page 42: CONTRACTS MNEMONICS · 2019. 9. 23. · ©2018 Pieper Bar Review 1 . CONTRACTS MNEMONICS . 1) The ingredients for a valid contract are TACO: T – Definite TERMS, express or implied

© 2018 Pieper Bar Review 40

20. A builder agreed to construct two cottages for an owner. One cottage was to have three bedrooms and the price agreed upon was $80,000. The other was to have two bedrooms and the price was $75,000. Upon completing each cottage, the owner was to pay the builder in full. The owner provided the plans and specifications, and told the builder that he was going to rent the houses for the summer months. The builder agreed to complete the two–bedroom cottage by March 1, and the three–bedroom cottage by March 15.

The builder began construction on the three bedroom cottage on January 5, and construction on the two–bedroom cottage on February 1. On February 20, after the three–bedroom cottage was three–fourths completed, it was destroyed by fire through the fault of neither the builder nor the owner. The builder completed the two–bedroom cottage on February 26.

If the builder refuses to rebuild the three–bedroom cottage, will he be able to recover on the contract for the completed two–bedroom cottage?

(A) Yes, because the builder did not

have an obligation to rebuild the three–bedroom cottage.

(B) Yes, only if the owner waives his claim for the breach of the contract for the three–bedroom cottage.

(C) Yes, because the contract is divisible.

(D) Yes, because of the doctrine of impossibility of performance.

20. Since the builder was to receive separate compensation for building each of the two houses, this was a divisible contract. Even if a party’s performance falls short of that required by the doctrine of substantial performance, a court can avoid forfeiture and allow recovery on the contract by holding that the contract is divisible (or severable) rather than entire. Under the doctrine of divisible contract, a party who is in breach can nevertheless recover for part performance.

Even if the builder does not build the $80,000 cottage, he has substantially performed his obligation to build the $75,000 cottage and so should be allowed to recover for that cottage under the contract. Thus, (C) is the correct answer.

Because the builder can recover for the $75,000 cottage regardless of his performance of the other part of the contract, (A), (B), and (D) are incorrect answers.

Note, the owner has a claim for damages against the builder for the builder’s failure to complete the three–bedroom cottage. Restatement (Second) of Contracts §240, illus. 1.

Page 43: CONTRACTS MNEMONICS · 2019. 9. 23. · ©2018 Pieper Bar Review 1 . CONTRACTS MNEMONICS . 1) The ingredients for a valid contract are TACO: T – Definite TERMS, express or implied

© 2018 Pieper Bar Review 41

21. A contractor entered into a written contract to construct a house for a homeowner in accordance with certain plans and specifications for $95,000, payable in installments as the work progressed. No lien law was applicable in the jurisdiction.

As the house neared completion and the contractor had been paid $90,000, the contractor informed the homeowner that his credit was shaky with plumbing supply company which sold him plumbing supplies. He explained that he would be unable to obtain delivery of $3,000 worth of sinks and toilets to complete the project, unless he paid the company $8,000 which he owed for the plumbing supplies previously purchased for the homeowner’s house.

The homeowner thereafter called the company on the telephone and told its manager that if he delivered the $3,000 worth of sinks and toilets which the contractor had ordered for the house, the homeowner would act as a surety for the contractor’s outstanding obligation to pay the $8,000.

The manager agreed and shipped the sinks and toilets. Thereafter, the contractor filed for bankruptcy without having made any payment to the plumbing supply and without completing the house for the homeowner.

If the plumbing supply company brings an action against the homeowner, will the plumbing supply company prevail?

(A) No, because the homeowner’s promise

to pay the $8,000 is unenforceable because of the statute of frauds.

(B) No, because the contractor was already obligated to pay for the plumbing supplies, and therefore the homeowner’s promise was not supported by consideration.

(C) Yes, and it will recover $3,000. (D) Yes, and it will recover $8,000.

21. A promise to pay the debt of another ordinarily must be in writing to be enforceable due to the statute of frauds. However, there is an exception under the “main purpose rule.” If the main purpose of the guarantee is to further the goals of the guarantor, the statute of frauds is inapplicable. That rule is applicable here, because the main purpose of the homeowner’s promise was to get his house finished. Restatement (Second) of Contracts §116 illus. 3 (1981). Therefore, (A) is incorrect.

(B) is incorrect because the manager’s shipment of the goods was valid consideration to support the homeowner’s promise to pay the debt.

The promise made by the homeowner was to pay the entire $8,000 due for plumbing materials used in the construction of his house. Once the main purpose rule takes the promise out of the statute of frauds, that promise is enforceable. Therefore, the homeowner owes the entire $8,000, not just the $3,000 for the fixtures shipped, and (D), not (C), is the correct answer.

Page 44: CONTRACTS MNEMONICS · 2019. 9. 23. · ©2018 Pieper Bar Review 1 . CONTRACTS MNEMONICS . 1) The ingredients for a valid contract are TACO: T – Definite TERMS, express or implied

© 2018 Pieper Bar Review 42

22. When is the court most likely to invoke the parol evidence rule?

(A) When it discovers evidence of fraud. (B) When parties seek to introduce

evidence concerning a consistent term in a fully integrated contract with a merger clause.

(C) When parties seek to introduce evidence concerning a consistent additional term on a partially integrated contract.

(D) When parties seek to introduce evidence explaining the trade meaning of a critical contract term in a fully integrated contract.

22. “A writing that is final is an integration of the terms embodied in it. When it is final and complete it is a total integration. A writing that is final, but does not completely express the parties’ contract is a partial integration.” Joseph M. Perillo, Contracts §3.2 at 113 (7th ed. 2014).

“Thus, a partial integration may not be contradicted by what has been called ‘parol evidence.’ A total integration not only cannot be contradicted by the type of evidence in question but cannot even be supplemental by consistent (non–contradictory) additional terms.” Id.; Restatement (Second) Contracts §210(1) and Comment a.

“A merger clause states that the writing is a final, complete, and exclusive statement of all of the terms agreed on. Williston’s first rule, which is followed by most courts, is that a merger clause will ordinarily resolve the issue of total integration.” Joseph M. Perillo, Contracts §3.6 at 128 (7th ed. 2014). Thus choice (B) is the best choice for invoking the parol evidence rule.

The general rule is that parol evidence is admissible to show fraud in the inducement. Thus, choice (A) is not correct.

A partial integration, being final but incomplete, may be supplemented by consistent additional terms.” Joseph M. Perillo, Contracts§ 3.2 at 114 (7th ed. 2014). Thus, the parol evidence rule would not be a bar to choice (C).

UCC 2–202 provides that even a writing intended by the parties as a final expression of their agreement may be explained or supplemented by a course of dealing, by trade usage or by a course of performance. Thus parol evidence could be used in choice (D).

Page 45: CONTRACTS MNEMONICS · 2019. 9. 23. · ©2018 Pieper Bar Review 1 . CONTRACTS MNEMONICS . 1) The ingredients for a valid contract are TACO: T – Definite TERMS, express or implied

© 2018 Pieper Bar Review 43

23. A restaurant owner and a contractor entered into an agreement for construction of a restaurant. The plans and specifications required the contractor to install an “Airflow Heating System.” About a week before the heating system was to be installed, the contractor discovered that it would take four months to obtain an “Airflow Heating System,” but that a “Flowheat Heating System” was immediately available. Since, the contractor was unable to contact the owner to receive his permission for the change, he consulted a heating engineer, who determined that the “Flowheat” system was as good as the “Airflow” system, it would heat just as well, and that it was of somewhat better quality. Relying upon this advice, the contractor installed the Flowheat system. The restaurant owner objected to the substitution of the Flowheat system and refused to pay the contractor.

In an action by the contractor against the restaurant owner for damages, which of the following would be the amount of damages recoverable by the contractor?

(A) The contractor can recover because

of the doctrine of impossibility of performance.

(B) The contractor can recover under the doctrine of substantial performance.

(C) The contractor cannot recover because he failed to contact the owner.

(D) The contractor can recover in quasi–contract.

23. (B) is correct because, under the doctrine of substantial performance, the contractor can recover on the contract. Although the contractor did not completely or exactly conform to the specifications of the contract, he substantially performed. Under the common law rule, substantial performance is enough to allow a party to recover on a contract, though he may still be liable for damages resulting from his technical breach. See Essay 3, July 2008.

To require the contractor to remove the “Flowheat” system and the cost of reinstalling an “Airflow” system would result in economic waste.

(A) is not applicable to this kind of situation. Generally, the doctrine of impossibility only applies to situations where the contract has become illegal, the subject matter of the contract has been destroyed, or a party to a personal services contract has died or become disabled.

(C) is incorrect because the owner was not available, so the contractor could not contact him.

(D) is incorrect because the doctrine of substantial performance allows recovery on the contract itself, not in quasi–contract.

Page 46: CONTRACTS MNEMONICS · 2019. 9. 23. · ©2018 Pieper Bar Review 1 . CONTRACTS MNEMONICS . 1) The ingredients for a valid contract are TACO: T – Definite TERMS, express or implied

© 2018 Pieper Bar Review 44

24. A renowned financial planner entered into a written contract with a woman to prepare an estate plan for a fee of $10,000, to be paid upon the completion of the plan. The woman retained the financial planner because of his reputation, skill and experience in financial planning. However, the woman and the financial planner got along poorly and, after the second conference, when only a draft of a plan dealing with life insurance and real estate had been submitted to the woman, they had a profound disagreement about the proper way to handle the woman’s stock portfolio. Hoping to avoid such encounters in the future, the financial planner, without the woman’s knowledge, assigned all of his rights and duties under his financial planning contract with the woman to an associate who had recently graduated from college. The associate expressly promised the financial planner to carry out the work to the best of his ability.

The woman, upon learning of the assignment, refused to allow the associate to proceed with the financial plan and brought an action against the financial planner to compel him to resume and complete performance of the contract.

Is the woman entitled to such relief?

(A) No, because the financial planner

has contracted to render personal services to the woman.

(B) No, because the financial planner effectively delegated his remaining duties under the contract to the associate.

(C) Yes, because the financial planner is a preeminent financial planner and his services are unique.

(D) Yes, because the financial planner has personally completed a substantial portion of the project.

24. The contract for an estate plan is a personal service, and so a specific performance remedy is not available. (A) is therefore correct.

(B) is incorrect because the financial planner cannot effectively delegate his obligations under a contract requiring his skill and expertise.

(C) is incorrect. Although it is the reason why the delegation to the associate was improper, it does not support an award of specific performance.

(D) is incorrect because the financial planner’s completion of a portion of the work does not become the basis of a specific performance remedy for the remainder.

Page 47: CONTRACTS MNEMONICS · 2019. 9. 23. · ©2018 Pieper Bar Review 1 . CONTRACTS MNEMONICS . 1) The ingredients for a valid contract are TACO: T – Definite TERMS, express or implied

© 2018 Pieper Bar Review 45

25. A seller agreed to sell 5,000 widgets to a buyer for $15,000. The goods were to be delivered on or before December 15. On December 5th, while the goods were in transit but before the goods were delivered, the buyer called the seller and told him that she did not want the goods and would not accept them because she had purchased them cheaper somewhere else.

What right did the buyer’s telephone call give the seller?

(A) The right to stop the goods in

transit if they were sent in carload lots.

(B) The right to stop the goods in transit even if they were not sent in carload lots.

(C) The right to stop delivery of the goods even if they had reached the buyer’s city and the carrier had acknowledged to the buyer that it was holding them for her.

(D) The right to stop the goods in transit only if the buyer is insolvent.

25. (A) is the correct answer because the buyer’s phone call constituted an anticipatory breach of the contract between the buyer and seller. Since this is a contract for the sale of goods, UCC Article 2 applies. Under Article 2, a seller has the right to stop goods in transit when there has been an anticipatory breach, but only if they were sent in carload lots (for example, an entire freight car of widgets). Therefore, (B) is incorrect.

(D) is incorrect because a seller has the right to stop the goods in transit upon the buyer’s anticipatory breach regardless of whether the buyer was insolvent. Furthermore, the facts do not indicate that the buyer was insolvent, but they do indicate that the buyer anticipatorily breached the contract. If the buyer was insolvent, the seller would have the right to stop the goods in transit whether or not they were sent in carload lots.

(C) is incorrect because a seller loses her right to stop goods in transit once a carrier has acknowledged the buyer’s right to the goods.

Page 48: CONTRACTS MNEMONICS · 2019. 9. 23. · ©2018 Pieper Bar Review 1 . CONTRACTS MNEMONICS . 1) The ingredients for a valid contract are TACO: T – Definite TERMS, express or implied

© 2018 Pieper Bar Review 46

26. A buyer and seller entered into a contract for the purchase and sale of 1,000 widgets at a price of $5 per widget. The widgets were to be shipped F.O.B. the buyer’s plant, and were to be delivered to the plant on or before December 1. The seller shipped them out on November 27. Later in the day on November 27, before the buyer received the widgets, the buyer called the seller and informed the seller that, since widgets were no longer popular gift items, he was cancelling the order. The goods were destroyed by fire after the buyer’s telephone call.

At the time of the fire, which party bore the risk of loss of the widgets?

(A) The buyer, because the risk of loss

shifted as soon as the seller delivered the goods to the carrier.

(B) The buyer, because she repudiated the contract.

(C) The buyer, but only to the extent that the seller’s insurance failed to cover the loss.

(D) The seller, because the contract specified “F.O.B. buyer’s plant.”

26. Ordinarily, the risk of loss passes entirely to the buyer when the seller completes his delivery obligations. Since the contract specifies F.O.B. buyer’s plant, the risk ordinarily would not pass until the goods actually reach the buyer. However, if the buyer repudiates or breaches the contract, the risk of loss immediately shifts to the buyer as to goods that are identified, to the extent that the seller’s insurance fails to cover the loss. UCC §2–510(1). Therefore, (C) is the correct answer, since the buyer repudiated the contract.

(A) is incorrect. Since the contract specifies F.O.B. buyer’s plant, the risk of loss did not shift when the seller delivered the goods to the carrier.

(B) is incorrect because the risk of loss only passed to the buyer to the extent that the seller’s insurance did not cover the loss.

(D) is incorrect because the risk of loss shifted to the buyer, in part, when he repudiated the contract.

Page 49: CONTRACTS MNEMONICS · 2019. 9. 23. · ©2018 Pieper Bar Review 1 . CONTRACTS MNEMONICS . 1) The ingredients for a valid contract are TACO: T – Definite TERMS, express or implied

© 2018 Pieper Bar Review 47

27. A manufacturer of off–road construction vehicles, entered into a written contract with a dealer to sell to the dealer three large trucks for $30,000 each. The manufacturer was to deliver one truck on March 1, one on April 1, and one on May 1. Payment for each of the trucks, in the amount of $30,000, was to be made within 5 days after each delivery. The manufacturer delivered the first truck on March 1.

On March 4, the dealer called the manufacturer and informed him that, because he had encountered a delay in obtaining a bank loan, he could not pay for the truck delivered on March 1 until March 20. The dealer, although he had been a reputable heavy equipment dealer for 25 years, had not paid for vehicles in a timely fashion since February 20. The dealer still has possession of the truck delivered on March 1.

Which of the following remedies is NOT available to the manufacturer on March 5?

(A) Immediately demanding a return of

the truck delivered on March 1, and reclaiming that truck.

(B) Refusing to deliver the truck scheduled for April 1 delivery unless the dealer pays for the first truck before then and pays for the April 1 truck in cash on or before the time of delivery.

(C) Suing the dealer for the $30,000. (D) Terminating the contract for the

April 1 and May 1 trucks.

27. The dealer was insolvent between February 20 and March 20 under the definition of insolvency under § 1–201 of the UCC because he could not pay his debts when they were due. Therefore, the manufacturer, after demanding the return of the truck within 10 days of delivery, could reclaim the truck delivered on March 1st under § 2–702 of the UCC. Therefore, (A) is an available remedy and an incorrect answer.

(B) is also an available remedy and, therefore, a wrong answer. If an insolvent buyer defaults on a payment obligation under an installment contract, the seller is not obligated to ship in the future on credit, but can demand payment in cash and can also demand that the insolvent buyer pay for any prior delivered installments before any further installments are delivered. UCC § 2–702(1).

(C) is also an available remedy and, therefore, a wrong answer. Where there is a breach of the obligation to pay for one installment, the seller can sue for that installment prior to completion of all deliveries, if the amount of each installment is clear, as it is in this case ($30,000 per installment).

(D) is the correct answer because the seller cannot terminate the entire installment contract when the buyer failed to pay only one installment on time, even when the buyer is technically insolvent. “An installment contract is one which requires or authorizes the delivery of goods in separate lots to be separately accepted....” UCC § 2–612.

The seller may, however, demand assurances of payment before he makes future deliveries and can require payment in cash at the time of delivery.

Page 50: CONTRACTS MNEMONICS · 2019. 9. 23. · ©2018 Pieper Bar Review 1 . CONTRACTS MNEMONICS . 1) The ingredients for a valid contract are TACO: T – Definite TERMS, express or implied

© 2018 Pieper Bar Review 48

28. A buyer and a seller entered into a written contract for the sale of 10,000 lawnmowers, for delivery on February 1. On January 10, the seller notified the buyer that due to a fire in its factory, the factory would remain closed until February 1, but the seller could guarantee delivery no later than February 15. The buyer rejected the seller’s proposal, immediately contacted with another lawnmower distributor to obtain 10,000 lawnmowers at a slightly higher cost, and brought an action against the seller to recover the difference between the cost of the replacement lawnmowers and the original contract price.

In the buyer’s action against the seller, will the buyer prevail?

(A) No, because the seller gave

assurances of its willingness and ability to perform the contract in February.

(B) No, because the buyer’s lawsuit is premature before February 1.

(C) Yes, because buyer’s complaint alleges an actionable tort by seller.

(D) Yes, because buyer’s complaint alleges an actionable breach of contract by seller.

28. The notice given by seller to buyer amounted to an anticipatory breach of contract by seller, since it unequivocally stated that it would not be able to ship conforming goods in the quantity stated at the time required. Such an anticipatory breach gives the buyer, among other remedies, the right to cover: i.e, the right to purchase the goods elsewhere and to sue the seller for the difference between the price the buyer had to pay for the goods and the original contract price. The buyer did this and now has a right to sue in contract for damages. Thus, (D) is the correct answer.

The buyer’s suit is based on breach of contract. The seller has committed no tort against the buyer. Therefore, (C) is incorrect.

(A) is incorrect because of the perfect tender rule under UCC Article 2. This rule does not allow for a unilateral delay in performance to a later, reasonable time when the seller has unforeseen difficulties.

(B) is incorrect because the buyer does not have to wait until the date of performance when there has been an anticipatory breach.

Page 51: CONTRACTS MNEMONICS · 2019. 9. 23. · ©2018 Pieper Bar Review 1 . CONTRACTS MNEMONICS . 1) The ingredients for a valid contract are TACO: T – Definite TERMS, express or implied

© 2018 Pieper Bar Review 49

29. On October 1, a buyer entered into a written contract with a seller for the purchase of 1,000 mechanical dogs at $20 per dog, to be specially manufactured by the seller according to the buyer’s specifications. In order to obtain operating funds, the seller, as borrower, entered into a written agreement on October 5 with a finance company. In relevant part, this agreement recited, “Seller hereby transfers and assigns to the finance company its October 1 mechanical dog contract with Buyer, as security for a 50–day loan of $15,000, the advance and receipt of which are hereby acknowledged by Seller ....” No copy of the agreement, or statement relating to it, was filed in an office of public record.

By November 16, the seller, without legal excuse, had delivered no dogs, but the buyer brought no action against the seller.

If the buyer brings an action against the finance company on account of seller’s default, will the buyer prevail?

(A) No, because the October 5

assignment by the seller to the finance company was only an assignment for security.

(B) No, because no record of the October 5 transaction between seller and finance company was publicly filed.

(C) Yes, because the buyer was a third–party intended beneficiary of the October 5 transaction between seller and the finance company.

(D) Yes, because the October 5 transaction between the seller and the finance company affected, with respect to buyer as creditor, a novation of debtors.

29. UCC 2–210 states that the assignee of a sale of goods contract impliedly assumes the full duties of performance under the terms of that contract unless it is clear from the circumstances of the assignment that the assignee did not intend to assume the delegation of duties.

The purpose of the assignment of the contract to the finance company was to give the finance company a security interest to insure repayment of the loan. It was only an assignment of the benefits of the contract, not a delegation of the duties. The finance company never expressly or impliedly agreed to perform the obligations of the contract and therefore, was not liable for a breach of the contract. Therefore, (A) is correct.

(B) is incorrect because the filing of the contract would only be material if there was a question of priority among competing security interests, a problem not present here. The critical issue with respect to the finance company’s liability is whether it accepted the obligations of the contract, which it did not.

(C) is incorrect because there was no intent to benefit the buyer in the security assignment made by the seller and the finance company, and so the buyer was not an intended third party beneficiary.

(D) is incorrect because there was no novation –– no agreement by the buyer, the finance company, and the seller to substitute the performance of the finance company for that of the sellers on the contract.

Page 52: CONTRACTS MNEMONICS · 2019. 9. 23. · ©2018 Pieper Bar Review 1 . CONTRACTS MNEMONICS . 1) The ingredients for a valid contract are TACO: T – Definite TERMS, express or implied

© 2018 Pieper Bar Review 50

30. A snowmobile manufacturer contracted with an engine company to purchase 100 engines for $250 each. The contract of sale expressly required that each engine would be able to generate 32 horsepower. The contract also provided that the engines would be delivered in lots of 25 and that payment would be made within 24 hours after delivery.

The first lot of 25 engines was delivered on June 1, at which time the snowmobile company forwarded payment to the engine company in the amount of $6,250. However, when later tested, none of the 25 engines delivered, or any of the 100 manufactured, could generate more than 15 horsepower.

Despite the lack of sufficient horsepower, the snowmobile company installed five of the engines in snowmobiles. On June 10, the snowmobile company rejected all of the engines that had been delivered, and advised that it would reject all future deliveries because of the insufficient horsepower.

On the above facts, how many engines did the snowmobile company accept?

(A) 0. (B) 5. (C) 25. (D) 100.

30. (B) is correct. The installation of five engines constitutes acceptance of those engines because it is a use which is inconsistent with the seller’s rights. The phrase “Despite the lack of sufficient horsepower” is a crucial clue showing that the buyer was aware that the goods were non–conforming yet elected to use five of them with knowledge of the non–conformity.

(A) is incorrect. The buyer used five of the engines with knowledge of a defect and that constituted acceptance.

(C) is incorrect. Each engine is a distinct commercial unit separable from the rest of the lot. A buyer may accept part of a lot and reject the rest, provided he does not break up a commercial unit. Here, the buyer accepted five units and properly rejected twenty. UCC § 2–601.

(D) is incorrect. The buyer has the right to accept part of the goods and reject the rest; hence, acceptance of five engines did not constitute acceptance of the total number specified in the contract.

Page 53: CONTRACTS MNEMONICS · 2019. 9. 23. · ©2018 Pieper Bar Review 1 . CONTRACTS MNEMONICS . 1) The ingredients for a valid contract are TACO: T – Definite TERMS, express or implied

© 2018 Pieper Bar Review 51

31. A manufacturer entered a contract to sell a $20,000 printer to a publisher. Just prior to the delivery date fixed in the contract, the manufacturer notified the publisher that it would be unable to deliver the machine.

Six months thereafter, the publisher purchased the same printer for $22,000. The publisher then brought a claim against the manufacturer for the publisher’s lost profits of $1,500 per month for the six months ($9,000) and the additional $2,000 the publisher had to pay to for the printer.

At the trial, the manufacturer introduced unrefuted credible evidence that if the publisher had purchased a replacement printer when the manufacturer breached the contract, the publisher could have readily purchased it for $20,000 and would have lost only one month of profit.

What is the maximum amount the printer is entitled to recover from the manufacturer?

(A) $2,000. (B) $11,000. (C) $1,500. (D) $3,500.

31. The traditional measure of damages for a total breach of contract by a seller is the difference between the market price of the goods at the time of the breach and the parties’ contract price. Under these facts, there are no breach of contract damages since the market price at the time of the contract’s breach ($20,000) was the same as the $20,000 contract price. Thus, neither (A), (B), nor (D) is correct.

The UCC also permits a buyer to “cover”, i.e., make a good faith purchase or enter a contract to purchase substitute goods without unreasonable delay. The buyer may then recover the difference between the cost of cover ($22,000) and the contract price ($20,000).

Here, however, waiting six months to cover is an unreasonable delay. The publisher’s “damages are measured as of the time the buyer could have covered.... [D]amages cannot be enhanced by the buyer’s remaining idle....” Joseph M. Perillo, Contracts §14.20 at 541 (7th ed. 2014).

Thus, the publisher can only recover $1,500 in consequential damages, i.e, up to the time the publisher could have covered. The publisher cannot recover for all six months of lost profit. Therefore, (B) would not be correct.

(C) is the correct choice. “[A] buyer who fails to cover may be denied consequential damages that could have been avoided by cover.” White & Summers, Uniform Commercial Code § 7–4 at 304 n.43 (6th ed. 2010).

Page 54: CONTRACTS MNEMONICS · 2019. 9. 23. · ©2018 Pieper Bar Review 1 . CONTRACTS MNEMONICS . 1) The ingredients for a valid contract are TACO: T – Definite TERMS, express or implied

© 2018 Pieper Bar Review 52

32. A coat manufacturer purchased 100 bolts of first quality wool from a wool producer. The sales contract provided that the manufacturer would remit payment prior to inspection. The wool was shipped, and the manufacturer paid the producer. Upon inspection, the manufacturer discovered that the wool was No. 2 quality. Thereupon, the manufacturer tendered back the wool to the producer and demanded return of his payment. The producer refused, contending there is no difference between No. 1 quality wool and No. 2 quality wool.

Which of the following statements regarding the contract provision concerning pre–inspection payment is correct?

(A) It constituted acceptance of the

goods. (B) It constituted a waiver of the

buyer’s remedy of private sale in the case of nonconforming goods.

(C) It did not impair the buyer’s right to inspect the goods prior to accepting or rejecting.

(D) It required the buyer to “accept” the goods without inspecting them, but did not affect the buyer’s right to revoke acceptance.

32. The basic rule is that a buyer always has the right to inspect goods before accepting them. This is true even if the buyer has agreed to pay for the goods prior to inspection. Remember to separate the buyer’s payment from the right of inspection.

(A) is incorrect because the mere agreement to pay before inspection in no way constitutes an acceptance of the goods. An acceptance, remember, occurs only when: (1) the buyer signifies that he or she is accepting; (2) the buyer uses the goods in some way inconsistent with the seller’s rights; or (3) the buyer fails to give timely notice of rejection. Therefore, (C) is the correct answer.

(B) is an absurd answer that is clearly incorrect. There is simply no connection between the promise to pay prior to inspection and the buyer’s remedy of selling the goods in a private sale.

(D) is also incorrect, as the clause does not require acceptance before inspection; it only requires payment.

Page 55: CONTRACTS MNEMONICS · 2019. 9. 23. · ©2018 Pieper Bar Review 1 . CONTRACTS MNEMONICS . 1) The ingredients for a valid contract are TACO: T – Definite TERMS, express or implied

© 2018 Pieper Bar Review 53

33. A publisher purchased 100 cases of white 24 lb. paper from a paper company. The publisher paid for the paper prior to receipt and the paper company sent 20 lb. paper instead of 24 lb. The publisher promptly tendered the paper back to the company and demanded return of its payment. The paper company refused, contending the papers to be identical.

Assuming that the 20 lb. paper is nonconforming, which of the following remedies is available to the publisher?

(A) Specific performance. (B) Damages, measured by the

difference between the value of the goods delivered and the value of conforming goods.

(C) Damages, measured by the price paid plus the difference between the contract price and the cost of buying substitute goods.

(D) None, since the publisher waived his remedies by agreeing to pay before inspection.

33. (A) is incorrect because specific performance is only allowed in sale of goods cases where the property is unique or in “other proper circumstances.” There is nothing unusual about these goods and nothing to indicate that damages would not afford a full remedy.

(B) is incorrect because this is the measure of damages to be used when the buyer accepts the goods. Here, however, the buyer rejected them.

(C) states the correct measure of damages because the publisher already paid for the goods and then rejected them as nonconforming. Thus, the publisher can recover restitution damages for the price paid plus the increased cost of cover.

(D) is incorrect because remedies are never waived by agreeing to pay for the goods.

Page 56: CONTRACTS MNEMONICS · 2019. 9. 23. · ©2018 Pieper Bar Review 1 . CONTRACTS MNEMONICS . 1) The ingredients for a valid contract are TACO: T – Definite TERMS, express or implied

© 2018 Pieper Bar Review 54

34. A shoe wholesaler received a catalogue from a shoe manufacturer which described all of the manufacturer’s shoes as “all leather except ornamentation” and “of the highest quality.”

Thereafter, the wholesaler and the manufacturer entered into an agreement whereby the manufacturer agreed to sell 10,000 pairs of shoes to the wholesaler.

The parties’ contract conspicuously stated:

“There are no warranties expressly or impliedly made by seller in connection with this sale beyond the description of the goods contained in the seller’s contract.”

Did the disclaimer in the contract negate any of the seller’s warranties?

(A) Yes, all express warranties. (B) Yes, the implied warranty of

fitness for a particular purpose. (C) Yes, the implied warranty of

merchantability. (D) No, the disclaimer was ineffective.

34. Choice (A) is incorrect because the catalogue clearly contains a description of the goods (“of the highest quality” is an express warranty). Where an express written warranty is given, a clause in the sales contract or the bill of sale purportedly disclaiming that warranty must fail. (C) is incorrect because the magic words “merchantability”, “as is,” or “with all faults” do not appear in the disclaimer clause, and hence the clause in the contract could not operate as a disclaimer of that warranty.

The disclaimer clause, however, will operate to negate any implied warranties of fitness, as a disclaimer of that warranty can be made by the use of general terminology. Therefore (B) is correct and (D) is incorrect.

Page 57: CONTRACTS MNEMONICS · 2019. 9. 23. · ©2018 Pieper Bar Review 1 . CONTRACTS MNEMONICS . 1) The ingredients for a valid contract are TACO: T – Definite TERMS, express or implied

UCC 2‐207 Contract Formation Charts 

For each of the following scenarios, assume that the buyer is a merchant (e.g. Diapers.com), 

sending an order form to the seller (e.g. Huggies) who is also a merchant, that the offer did not 

expressly condition acceptance upon the terms of the offer, and that the buyer did not object 

to the terms of the seller’s acknowledgment form within a reasonable time. 

Article 2 Acceptance Containing: 

 Additional Terms That Do Not Materially Alter the Offer 

Diapers.comOffer/Order Form

•Item:  Size 5 OvernightDiapers

•Quantity:  100 Cases

•Price:  $100 per case

•Delivery:  Buyer's Warehouse, 01/1/17

HuggiesAcceptance/

Acknowledgment

•Item:  Size 5 OvernightDiapers

•Quantity:  100 Cases

•Price:  $100 per case

•Delivery:  Buyer's Warehouse, 1/1/17•Buyer shall pay 2% intereston all overdue invoices

Resulting Contract?

•Yes, the seller'sacknowledgement form is adefinite and seasonableexpression of acceptance, sothe parties have a contractdespite the addition of a termcharging interest for overdueinvoices in theacknowledgment form.  Thecontract consists of all of theterms in the acceptance,including the term providingfor reasonable interest onoverdue invoices, as suchterms are common and dowould not cause surprise orhardship on the offeror.

•Note that, at common law,the acceptance containing anadditional term would havebeen deemed a counteroffer,and thus there would be nobinding contract upon theseller's acceptance unless thebuyer accepted thecounteroffer, either expresslyor through physicalacceptance of the goods.

Page 58: CONTRACTS MNEMONICS · 2019. 9. 23. · ©2018 Pieper Bar Review 1 . CONTRACTS MNEMONICS . 1) The ingredients for a valid contract are TACO: T – Definite TERMS, express or implied

UCC 2‐207 Contract Formation Charts 

Article 2 Acceptance Containing:  

Additional Terms that Materially Alter the Offer 

Diapers.comOffer/Order Form

•Item:  Size 5 Overnight Diapers

•Quantity:  100 Cases

•Price:  $100 per case

•Delivery:  Buyer's Warehouse, 1/1/17

HuggiesAcceptance/

Acknowledgment

•Item:  Size 5 Overnight Diapers

•Quantity:  100 Cases

•Price:  $100 per case

•Delivery:  Buyer's Warehouse, 1/1/17•All Disputes Between theParties Shall be ResolvedThrough Binding Arbitration

Resulting Contract?

•Yes, the seller'sacknowledgement form is adefinite and seasonableexpression of acceptance, sothe parties have a contractdespite the addition of anarbitration clause in theacknowledgment form.However, since the arbitrationclause would take the offerorby surprise and cause theofferor hardship, it would"materially alter" the offer andnot become part of theparties' agreement.  Thecontract would consist of allthe terms except thearbitration clause.

•Note that if it were thecustom in the diaper industryto resolve disputes througharbitration (as is the custom,for example, in the steel andtextile industries), then theaddition of an arbitrationclause could not be said totake the offeror by surpriseand the term would not bedeemed a material alterationof the offer (thus becomingpart of the parties contract).

•Also  note that, at commonlaw, the acceptancecontaining an additional termwould have been deemed acounteroffer, and thus therewould be no binding contractupon the seller's acceptanceunless the buyer accepted thecounteroffer, either expresslyor through physicalacceptance of the goods.

Page 59: CONTRACTS MNEMONICS · 2019. 9. 23. · ©2018 Pieper Bar Review 1 . CONTRACTS MNEMONICS . 1) The ingredients for a valid contract are TACO: T – Definite TERMS, express or implied

UCC 2‐207 Contract Formation Charts 

Article 2 Acceptance Containing: 

 Different Terms  

Diapers.comOffer/Order Form

•Item:  Size 5 Overnight Diapers

•Quantity:  100 Cases

•Price:  $100 per case

•Delivery:  Buyer's Warehouse, 1/1/17•The parties agree that all actions orproceedings arising in connectionwith this Agreement shall be triedand litigated exclusively in the Stateand Federal courts located in theCounty of New York, State of NewYork.

HuggiesAcceptance/

Acknowledgment

•Item:  Size 5 Overnight Diapers

•Quantity:  100 Cases

•Price:  $100 per case

•Delivery:  Buyer's Warehouse, 1/1/17•The parties agree that all actions orproceedings arising in connectionwith this Agreement shall be triedand litigated exclusively in the Stateand Federal courts located in theCounty of San Francisco, State ofCalifornia.

Resulting Contract?

•Yes, the seller's acknowledgementform is a definite and seasonableexpression of acceptance, so theparties have a contract despite thedifferent forum selection clause inthe acknowledgment form.  Underthe majority position, the contractwould consist of those the terms onwhich the offer and acceptanceagreed, but the forum selectionclauses would "Drop Out" of theparties' contract.  Since there is noUCC provision regarding forumselection, normal civil procedurerules would apply in determiningwhere the action or proceedingwould take place.  In New York,courts would reach a differentconclusion, however, treating the"different" forum selection clause asif it was an "additional" term under2‐207(2).  Since the different forumselection clause would take theofferor by surprise and cause theofferor hardship, it would "materiallyalter" the offer and not become partof the parties' agreement.  Thecontract would consist of the termsin the buyer's offer, including thebuyer's forum selection clause.

•Note that, at common law, theacceptance containing a differentterm would have been deemed acounteroffer, thus there would be nobinding contract upon the seller'sacceptance unless the buyeraccepted the counteroffer, either expressly or through physicalacceptance of the goods.

Page 60: CONTRACTS MNEMONICS · 2019. 9. 23. · ©2018 Pieper Bar Review 1 . CONTRACTS MNEMONICS . 1) The ingredients for a valid contract are TACO: T – Definite TERMS, express or implied

UCC 2‐207 Contract Formation Charts 

Article 2 Acceptance Containing: 

 Different PDQ Terms 

Diapers.comOffer/Order Form

•Item:  Size 5 OvernightDiapers

•Quantity:  100 Cases

•Price:  $100 per case

•Delivery:  Buyer'sWarehouse, 1/1/17

HuggiesAcceptance/

Acknowledgment

•Item:  Size 5 OvernightDiapers

•Quantity:  100 Cases

•Price:  $110 per case

•Delivery:  Buyer'sWarehouse, 1/1/17

Resulting Contract?

•No, because the seller'sacknowledgement formcontained a different price(PDQ) term, and thereforecannot be considered adefinite expression ofacceptance.   Therefore,neither party may be foundliable for breaching thisattempted contract if adispute arises before eitherparty partially or fullyperforms.

•Note that, at common law,the acceptance containing adifferent term would havebeen deemed acounteroffer, and thus therewould be no bindingcontract upon the seller'sacceptance unless the buyeraccepted the counteroffer,either expressly or throughphysical acceptance of thegoods.

Page 61: CONTRACTS MNEMONICS · 2019. 9. 23. · ©2018 Pieper Bar Review 1 . CONTRACTS MNEMONICS . 1) The ingredients for a valid contract are TACO: T – Definite TERMS, express or implied

UCC 2‐207 Contract Formation Charts 

Article 2 Acceptance Containing: 

Different PDQ Terms,  

But Conduct of the Parties Recognizes the Existence of a Contract 

Diapers.comOffer/Order Form

•Item:  Size 5Overnight Diapers

•Quantity:  100 Cases

•Price:  $100 per case

•Delivery:  Buyer's Warehouse, 1/1/17

HuggiesAcceptance/

Acknowledgment

•Item:  Size 5Overnight Diapers

•Quantity:  100 Cases

•Price:  $110 per case

•Delivery:  Buyer's Warehouse, 1/1/17

Resulting Contract?

•No, because theseller'sacknowledgementform contained adifferent price (PDQ)term and, therefore,cannot be considereda definite expressionof acceptance.Therefore, neitherparty may be heldliable for breachingthis attemptedcontract if a disputearises before eitherparty partially or fullyperformed.

•Note that, atcommon law, theacceptancecontaining a differentterm would havebeen deemed acounteroffer, thusthere would be nobinding contract uponthe seller'sacceptance unless thebuyer accepted thecounteroffer, eitherexpressly or throughphysical acceptanceof the goods.

Conduct By Both Parties Recognizes the Existence of a Contract

•However, if the buyerphysically acceptedthe 100 cases ofdiapers and beganselling them despitethe fact that the offerand acceptancecontained a differentPDQ term, the Codewould recognize theexistence of acontract through theconduct of the parties(even though thewritings did nototherwise establish acontract).  The termsof that contractwould consist ofthose terms on whichthe offer andacceptance agree, thedifferent price termswould drop out, andUCC 2‐305 wouldserve as a gap‐fillerimposing "areasonable price atthe time for delivery."

Page 62: CONTRACTS MNEMONICS · 2019. 9. 23. · ©2018 Pieper Bar Review 1 . CONTRACTS MNEMONICS . 1) The ingredients for a valid contract are TACO: T – Definite TERMS, express or implied

UCC 2‐207 Contract Formation Charts 

Below is the text of section 2‐207, which may help you understand why there is disagreement 

among  legal  scholars  and  courts  as  to  how  “different”  terms  contained  in  a  definite  and 

seasonable expression of acceptance should be treated.   Look,  in particular, at how section 2‐

207(1)  states  how  a  definite  and  seasonable  expression  of  acceptance  containing  either 

“additional”  or  “different”  terms  can  create  a  contract.    Then  notice  how  section  2‐207(2) 

addresses how to treat “additional” terms.  However, section 2‐207(2) says nothing about how 

“different” terms should be treated, and there are no other sections of the Code on point.  New 

York essentially takes the position that the drafters made a mistake and meant to have 2‐207(2) 

apply to “additional or different” terms.  The majority of courts, however, apply the knock out 

rule  to different  terms.   That  is,  the different  terms cancel each other out, and  the  resulting 

contract consists of those terms on which the offer and acceptance agree, as well as any relevant 

“gap fillers” under the Code (similar to an analysis under 2‐207(3) under the Code). 

Section 2‐‐207.  Additional Terms in Acceptance or Confirmation. 

(1) A definite and seasonable expression of acceptance or a written confirmation which is sent within a reasonable time operates as an acceptance even though it states terms additional to or different from those offered or agreed upon, unless acceptance is expressly made conditional on assent to the additional or different terms. 

(2) The additional terms are to be construed as proposals for addition to the contract.  Between merchants such terms become part of the contract unless: (a) the offer expressly limits acceptance to the terms of  the offer; (b) they materially alter it; or (c)  notification of objection to them has already been given or is given within a 

reasonable time after notice of them is received. (3) Conduct by both parties which recognizes the existence of a contract is sufficient to 

establish a contract for sale although the writings of the parties do not otherwise establish a contract.  In such case the terms of the particular contract consist of those terms on which the writings of the parties agree, together with any supplementary terms incorporated under any other provisions of this Act. 

Page 63: CONTRACTS MNEMONICS · 2019. 9. 23. · ©2018 Pieper Bar Review 1 . CONTRACTS MNEMONICS . 1) The ingredients for a valid contract are TACO: T – Definite TERMS, express or implied

UCC 2‐207 Contract Formation Charts 

Was there a "definite" and "seasonable" expression of acceptance?

Yes.

Were there any additional or different terms in the acceptance?

Yes.

Was the acceptance expressly conditioned on assent to the additional  or different terms in the acceptance?

Yes. 

A contract was not formed (unless the offeror expressly assented to the additional terms).

No.

A contract may have been formed.  How should you treat additional or different 

terms in the acceptance?

Different terms.

Was the offer expressly conditioned upon assent to its terms?

No.

Did the different 

terms relate to PDQ?

Yes.

The parties do not have a contract 

because there there can be no "definite" acceptance with different PDQ terms.

No.

The parties have a contract consiting of those terms on 

which the offer and acceptance agree.  The 

different terms "drop out" and are replaced by any  

relevant "gap fillers" which the Code may or may not 

provide. 

Yes.

A contract was formed based on the offeror's terms and the different terms are mere 

proposals.

If the dispute is under NY law, return to the additional 

terms analysis.

Additional Terms.

Was the offer expressly conditioned on assent to 

its terms?

No.

Are both parties merchants?

No.

The parties have a contract. However, 

between non‐merchants, 

additional terms contained in an acceptance are treated as mere proposals that 

must be expressly 

accepted by the offeror before becoming part of the parties' agreement.

Yes.

Did the offeror object to the additional term within a reasonable time

Yes.

The parties have a contract 

without the additional term.

No.

Did the additional term "materially alter" the 

offer?

No.

The parties have a contract 

and the additional term 

in the acceptance is included in the 

contract.

Yes.

The parties have a contract without the additional term.

Yes.

A contract was formed based on the offeror's terms and the additional 

terms are mere proposals.

No.

A contract was formed based on the offeror's 

terms.

No.

No contract was formed.

Page 64: CONTRACTS MNEMONICS · 2019. 9. 23. · ©2018 Pieper Bar Review 1 . CONTRACTS MNEMONICS . 1) The ingredients for a valid contract are TACO: T – Definite TERMS, express or implied

Risk of Loss Scenarios 

The UCC puts risk of loss on the party who is in the best position to bear that risk (i.e. the party who most 

likely has  taken precautions  to protect  the goods  from  loss by  insuring  them).   The Code  lists several 

scenarios that identify when risk of loss transfers from a seller to a buyer. 

1) When the goods are held by a bailee (e.g., a warehouse) and the goods are to be delivered to thebuyer without being moved or shipped, then risk of loss passes to the buyer a reasonable timeafter a buyer’s receipt of a bill of lading or the bailee otherwise notifying the buyer of the buyer’sright to take possession of the goods (that is, the buyer has a reasonable time to get the goodsunder 2‐509(2)(c)).  A “bill of lading” is simply a document issued by a carrier acknowledging itsreceipt of certain goods, which typically identifies the seller/sender of the goods, and to whom, ifanyone, the goods should be delivered.  The bill of lading is, in essence, a receipt for the goods, acontract for shipping the goods, and documentary evidence of title to goods.  If a seller negotiatesa carrier’s bill of lading to a buyer, then the buyer has title to the goods, such that the carrier mustturn over the goods upon the buyer’s presentation of the bill of lading.  So if the seller ships itsgoods by a carrier to a third‐party warehouse, then receives payment from the buyer, and turnsover the bill of lading to the buyer allowing the buyer to take possession of the goods, risk of losswill transfer from the seller to the buyer once the buyer has had a reasonable time to get thegoods from the warehouse (since the buyer has title at this point, the reasonable time will not bevery long, rather, it will be just enough time to allow the buyer to take possession of the goodsand/or insure them).

2) Under a “shipment” contract (where the goods are to be delivered to the buyer, but the contractdoes  not  require  the  seller  to  do  anything  other  than make  arrangements  for  shipping  by  acarrier), risk of  loss transfers to the buyer when the seller delivers the goods to a carrier.   Forexample, a contract that is F.O.B. seller’s place of business.

3) Under  a  “destination”  contract  (where  the  seller  is  required  to  deliver  goods  to  a  particulardestination),  risk  of  loss  transfers  to  the  buyer when  the  goods  are  tendered  to  the  buyer,enabling the buyer  to take possession.   For example, a contract that  is F.O.B. buyer’s place ofbusiness.

4) When the buyer is to pick up the goods from a merchant’s place of business, then risk of loss doesnot pass to the buyer until the buyer takes possession of the goods.  If the seller is not a merchant,

Seller Buyer

Page 65: CONTRACTS MNEMONICS · 2019. 9. 23. · ©2018 Pieper Bar Review 1 . CONTRACTS MNEMONICS . 1) The ingredients for a valid contract are TACO: T – Definite TERMS, express or implied

Risk of Loss Scenarios 

then  risk of  loss passes upon  tender of delivery.   For example,  if you purchase goods  from a merchant (in class we used the example of a suit), and elect to pick them up at a later date, then risk of loss does not pass to you (the buyer) until you take possession of the goods. 

Note that there is a limitation on recovering damages when specific goods have been identified 

and are totally destroyed through no fault of either party before risk of loss passes to the buyer.  

With the goods being destroyed, the Code impliedly rescinds the contract allowing the buyer to 

recover any deposit (restitution damages), but the buyer cannot assert a claim for expectation 

damages for the buyer’s lost bargain.  So, if we refer back to the example from class, in which the 

buyer purchased two $1,000 suits from the merchant seller for a sale price of $100 each, the buyer 

paid for the suits but elected to pick them up at a later date, and the suits were destroyed in a fire 

at the merchant’s store, the risk of loss would still be on the seller since the buyer had not taken 

possession of the suits, but the buyer could only recover the $200 he paid for the suits, not the 

value of his lost bargain (the $2,000 value of the suits he would have received had suits not been 

destroyed).  

5) When a seller ships non‐conforming goods, then risk of loss remains with the seller until cure oracceptance of the non‐conforming goods.  For example, even if a buyer and seller have entered acontract that  is F.O.B. seller’s place of business  (where risk of  loss would normally pass to thebuyer when the seller delivers the goods to a carrier, number 2 above),  if the seller ships non‐conforming goods and the goods are destroyed while in the carrier’s possession, the risk of losswould still be on the seller because the goods were non‐conforming.

6) If after acceptance, the buyer properly revokes acceptance, the risk of loss is on the seller to theextent of any deficiency  in the buyer’s  insurance.   For example,  if computers purchased by thebuyer were fitted with faulty processors that the buyer could not have noticed upon inspection(they were a latent defect), and the buyer revokes acceptance, but the goods are destroyed priorto the exchange of the goods for a return of the purchase price, the seller will bear the risk of lossto the extent of any deficiency  in the buyer’s  insurance.   Note that a buyer cannot attempt torevoke acceptance after  the goods have been  lost or destroyed  (that  is,  the buyer must havenotified the seller of his intention to revoke acceptance prior to the loss of the goods).

7) When conforming goods are identified to the contract and the buyer repudiates before risk of losshas  passed  to  the  buyer,  then  for  a  commercially  reasonable  time  the  risk  of  loss  is  on  thebreaching buyer to the extent of any deficiency in the seller’s insurance.  For example, consider acontract that required the seller to manufacture 40,000 pounds of plastic pellets and to send themto the buyer at a rate of 1,000 per day.   After manufacturing  the pellets, the seller requestedinstructions regarding where to begin shipping the pellets, but the buyer sent no  instructions.After storing the pellets for over 40 days, the seller sent a letter to the buyer explaining that itcould not store the pellets indefinitely and requesting immediate shipping instructions.  The sellerclaimed that the instructions would be forthcoming, but they never came, and within a month,the goods were destroyed in an accidental fire at the warehouse.  In this scenario, a court foundthe buyer had repudiated the contract by not sending the shipping instructions under the termsof the contract, and found that the risk of loss was on the buyer to the extent the seller’s insurancecoverage on the 40,000 pellets had expired.

Page 66: CONTRACTS MNEMONICS · 2019. 9. 23. · ©2018 Pieper Bar Review 1 . CONTRACTS MNEMONICS . 1) The ingredients for a valid contract are TACO: T – Definite TERMS, express or implied

Sample Bill of Lading 

10